[R] Splitting a vector into equal groups

2009-05-04 Thread utkarshsinghal
Hi All,

I have vector of length 52, say, x=sample(30,52,replace=T). I want to 
sort x and split into five *nearly equal groups*. Note that the 
observations are repeated in x so in case of a tie I want both the 
observations to fall in same group.
This seems a very common task to do, but still I couldn't find an R 
function to do this. Any help would be highly appreciated.

Regards
Utkarsh



[[alternative HTML version deleted]]

__
R-help@r-project.org mailing list
https://stat.ethz.ch/mailman/listinfo/r-help
PLEASE do read the posting guide http://www.R-project.org/posting-guide.html
and provide commented, minimal, self-contained, reproducible code.


Re: [R] Splitting a vector into equal groups

2009-05-04 Thread ronggui
lattice:::equal.count may be what you want.

2009/5/4 utkarshsinghal utkarsh.sing...@global-analytics.com:
 Hi All,

 I have vector of length 52, say, x=sample(30,52,replace=T). I want to
 sort x and split into five *nearly equal groups*. Note that the
 observations are repeated in x so in case of a tie I want both the
 observations to fall in same group.
 This seems a very common task to do, but still I couldn't find an R
 function to do this. Any help would be highly appreciated.

 Regards
 Utkarsh



        [[alternative HTML version deleted]]

 __
 R-help@r-project.org mailing list
 https://stat.ethz.ch/mailman/listinfo/r-help
 PLEASE do read the posting guide http://www.R-project.org/posting-guide.html
 and provide commented, minimal, self-contained, reproducible code.




-- 
HUANG Ronggui, Wincent
PhD Candidate
Dept of Public and Social Administration
City University of Hong Kong
Home page: http://asrr.r-forge.r-project.org/rghuang.html

__
R-help@r-project.org mailing list
https://stat.ethz.ch/mailman/listinfo/r-help
PLEASE do read the posting guide http://www.R-project.org/posting-guide.html
and provide commented, minimal, self-contained, reproducible code.


Re: [R] Splitting a vector into equal groups

2009-05-04 Thread Dimitris Rizopoulos
check functions cut() and quantile(), and cut2() from package Hmisc; 
maybe the following is close to what you want:


x - sample(30, 52, replace = TRUE)

k - 5 # how many groups
qs - quantile(x, seq(0, 1, length.out = k + 1))
y - cut(x, round(qs), include.lowest = TRUE)
y
table(y)


I hope it helps.

Best,
Dimitris


utkarshsinghal wrote:

Hi All,

I have vector of length 52, say, x=sample(30,52,replace=T). I want to 
sort x and split into five *nearly equal groups*. Note that the 
observations are repeated in x so in case of a tie I want both the 
observations to fall in same group.
This seems a very common task to do, but still I couldn't find an R 
function to do this. Any help would be highly appreciated.


Regards
Utkarsh



[[alternative HTML version deleted]]

__
R-help@r-project.org mailing list
https://stat.ethz.ch/mailman/listinfo/r-help
PLEASE do read the posting guide http://www.R-project.org/posting-guide.html
and provide commented, minimal, self-contained, reproducible code.



--
Dimitris Rizopoulos
Assistant Professor
Department of Biostatistics
Erasmus University Medical Center

Address: PO Box 2040, 3000 CA Rotterdam, the Netherlands
Tel: +31/(0)10/7043478
Fax: +31/(0)10/7043014

__
R-help@r-project.org mailing list
https://stat.ethz.ch/mailman/listinfo/r-help
PLEASE do read the posting guide http://www.R-project.org/posting-guide.html
and provide commented, minimal, self-contained, reproducible code.


Re: [R] Splitting a vector into equal groups

2009-05-04 Thread Berwin A Turlach
G'day Utkarsh,

On Mon, 04 May 2009 11:51:21 +0530
utkarshsinghal utkarsh.sing...@global-analytics.com wrote:

 I have vector of length 52, say, x=sample(30,52,replace=T). I want to 
 sort x and split into five *nearly equal groups*.

What do you mean by *nearly equal groups*?  The size of the groups
should be nearly equal? The sum of the elements of the groups should be
nearly equal?

 Note that the observations are repeated in x so in case of a tie I
 want both the observations to fall in same group.

Then it becomes even more important to define what you mean with
nearly equal groups.

As a start, you may consider:

R set.seed(1)
R x=sample(30,52,replace=T)
R xrle - rle(sort(x))
R xrle
Run Length Encoding
  lengths: int [1:25] 2 1 2 2 3 1 1 1 5 1 ...
  values : int [1:25] 1 2 4 6 7 8 9 11 12 13 ...
R cumsum(xrle$lengths)
 [1]  2  3  5  7 10 11 12 13 18 19 24 25 26 28 29 32 35 38
[19] 43 45 46 48 49 51 52

and use this to determine our cut-offs.  E.g., should the first group
have 10, 11 or 12 elements in this case?  The information in xrle
should enable you to construct your five groups once you have decided
on a grouping.

HTH.

Cheers,

Berwin

=== Full address =
Berwin A TurlachTel.: +65 6516 4416 (secr)
Dept of Statistics and Applied Probability+65 6516 6650 (self)
Faculty of Science  FAX : +65 6872 3919   
National University of Singapore 
6 Science Drive 2, Blk S16, Level 7  e-mail: sta...@nus.edu.sg
Singapore 117546http://www.stat.nus.edu.sg/~statba

__
R-help@r-project.org mailing list
https://stat.ethz.ch/mailman/listinfo/r-help
PLEASE do read the posting guide http://www.R-project.org/posting-guide.html
and provide commented, minimal, self-contained, reproducible code.


[R] QUADRATIC TREND FOR LINK FUNCTIONS ON NON-STATIONARY GEV

2009-05-04 Thread albertob

Hi All,
I am a newcomer to R. Could anyone explain me how to define link functions
for either mu/sigma to allow for quadratic trends in the same, when fitting
non-stationary GEV distributions?
Thanks
-- 
View this message in context: 
http://www.nabble.com/QUADRATIC-TREND-FOR-LINK-FUNCTIONS-ON-NON-STATIONARY-GEV-tp23360751p23360751.html
Sent from the R help mailing list archive at Nabble.com.

__
R-help@r-project.org mailing list
https://stat.ethz.ch/mailman/listinfo/r-help
PLEASE do read the posting guide http://www.R-project.org/posting-guide.html
and provide commented, minimal, self-contained, reproducible code.


[R] Question of Quantile Regression for Longitudinal Data

2009-05-04 Thread Helen Chen

Dear R user,

I am trying to estimate a quantile regression using panel data. I  
am trying to use the model that is described in Dr. Koenker's article. So I  
use the code the that is posted in the following link:

 http://www.econ.uiuc.edu/~roger/research/panel/rq.fit.panel.R

I use plm package to estimate OLS estimators.
Then I ploted the figure with  Quantile estimators and OLS estimators
But the line of Quantile estimators  was not cross  the line of  OLS
estimators.
I don't know why. 
 

OLS estimator is  0.00108194

Quantile estimators with 9 quantiles(0.1 to 0.9) as follow:
  0.0002071648  , 0.0002071899 , 0.0002149157 , 0.0002242560 , 0.0002254730
  0.0002404203 0.000232909 0.0002511185 0.0003015694


 Thanks
 I really would appreciate some suggestions.
 Best
 Helen Chen

-- 
View this message in context: 
http://www.nabble.com/Question-of-%22Quantile-Regression-for-Longitudinal-Data%22-tp23363232p23363232.html
Sent from the R help mailing list archive at Nabble.com.

__
R-help@r-project.org mailing list
https://stat.ethz.ch/mailman/listinfo/r-help
PLEASE do read the posting guide http://www.R-project.org/posting-guide.html
and provide commented, minimal, self-contained, reproducible code.


[R] normality test for large a large dataset ?

2009-05-04 Thread Martial Sankar


Hello, 

Do you know a R implemented normality test like the shapiro test but more 
suitable for large data set ?

Thanks,



_
Découvrez toutes les possibilités de communication avec vos proches

[[alternative HTML version deleted]]

__
R-help@r-project.org mailing list
https://stat.ethz.ch/mailman/listinfo/r-help
PLEASE do read the posting guide http://www.R-project.org/posting-guide.html
and provide commented, minimal, self-contained, reproducible code.


Re: [R] Division ?

2009-05-04 Thread Martin Maechler
 TH == Ted Harding ted.hard...@manchester.ac.uk
 on Sun, 03 May 2009 10:11:04 +0100 (BST) writes:

TH On 02-May-09 17:34:15, bogdanno wrote:
 It seems division with numbers bigger than 10 000 000
 doesn't work 2000/21 [1] 952381
 /23
 [1] 2415459
 
 Thank you

TH I think you are confusing what is displayed with what is
TH computed:

TH   2000/21 # [1] 952381 print(2000/21,17) # [1]
TH 952380.9523809524

TH   /23 # [1] 2415459 print(/23,17) # [1]
TH 2415458.913043478

TH   (2000/21)*21 # [1] 2e+07
TH print((2000/21)*21,17) # [1] 2e+07

TH   (/23)*23 # [1] 
TH print((/23)*23,17) # [1] 

TH Your numbers bigger than 10 000 000 corresponds to the
TH default display of results to 7 significant figures.

TH If (as in the above print() statements) you increase
TH this, you get more reasonable-looking results.

Yes, indeed.
If Bogdanno thinks he wants to always see all digits, he
should use something like

  options(digits= 20)

in his R startup setting, e.g. in his Rprofile, see  ?Startup.

Note however:  
 Quite a few experienced R users rather do  *REDUCE* the default number
 of digits printed, rather than enlarge them.
 (I don't)

Martin Maechler, ETH Zurich


TH 
TH E-Mail: (Ted Harding) ted.hard...@manchester.ac.uk
TH Fax-to-email: +44 (0)870 094 0861 Date: 03-May-09 Time:
TH 10:11:02 -- XFMail
TH --

__
R-help@r-project.org mailing list
https://stat.ethz.ch/mailman/listinfo/r-help
PLEASE do read the posting guide http://www.R-project.org/posting-guide.html
and provide commented, minimal, self-contained, reproducible code.


[R] quadratic programming

2009-05-04 Thread Barbara . Rogo

Devo risolvere un problema di minimo vincolato con vincoli di uguaglianza e un 
altro con vincoli di uguaglianza e disuguaglianza.
Cosa posso utilizzare?
[[alternative HTML version deleted]]

__
R-help@r-project.org mailing list
https://stat.ethz.ch/mailman/listinfo/r-help
PLEASE do read the posting guide http://www.R-project.org/posting-guide.html
and provide commented, minimal, self-contained, reproducible code.


[R] Surface for R outside of R

2009-05-04 Thread koj

Hi all,

i am new here and this is my first posting. My coding experience in R ist
okay, but I have a problem which is related to another aspect of R.

I am searching for a surface which execute my commands in R via a click on a
button OUTSIDE of R. Concrete: I will write some programs in R and want to
execute this files outside of R, so I don't want to start R eyery time if i
want an analysis. The best case were a surface of e.g. 8 Buttons, each click
leads to start a specific R file. My outputs are JPEG or CSV, so I don`t
need the output inside of R. Could anyone can give me some recommendations,
what could be a solution (e. g. Java)? Is such a solution possible? 

Thank you very much,

Jens.
-- 
View this message in context: 
http://www.nabble.com/Surface-for-R-outside-of-R-tp23364631p23364631.html
Sent from the R help mailing list archive at Nabble.com.

__
R-help@r-project.org mailing list
https://stat.ethz.ch/mailman/listinfo/r-help
PLEASE do read the posting guide http://www.R-project.org/posting-guide.html
and provide commented, minimal, self-contained, reproducible code.


[R] Need to clean a table, and compute mean and SD

2009-05-04 Thread Thom_249

Hi

For a school project I have a file with 120 columns and  ~2000 lines. This
file contains timestamps of spike detected in 60 channels, and the time
elapsed between the last spike.

I need to clean too high values. About 98% of values are between 0 and 2000
and 2% are between 2000 and 20'000. I want to get rid of theses values.

Please could you help me?

Regards 

Thom
-- 
View this message in context: 
http://www.nabble.com/Need-to-clean-a-table%2C-and-compute-mean-and-SD-tp23364460p23364460.html
Sent from the R help mailing list archive at Nabble.com.

__
R-help@r-project.org mailing list
https://stat.ethz.ch/mailman/listinfo/r-help
PLEASE do read the posting guide http://www.R-project.org/posting-guide.html
and provide commented, minimal, self-contained, reproducible code.


[R] About the Transfer Function Model(ARIMAX)

2009-05-04 Thread Ramanath Roy
Dear ALL,

I would appreciate if someone help me by letting me know the code of above 
model in R.I would request you to please let me know how i could 
make arimax model in auto.arima.


Regards

Ramanath
[[alternative HTML version deleted]]

__
R-help@r-project.org mailing list
https://stat.ethz.ch/mailman/listinfo/r-help
PLEASE do read the posting guide http://www.R-project.org/posting-guide.html
and provide commented, minimal, self-contained, reproducible code.


Re: [R] How to skip rest of code

2009-05-04 Thread bartjoosen

How about:

x - F
if (x) {
 print(true) 
} else {
print(false)
}


Bart


mli-2 wrote:
 
 Dear R users,
  
 Suppose I have 2 R script files: 'test1.R' and 'test2.R' and one R file
 'main.R' which sources each of them. I wonder if there is a way to skip
 rest of code in 'test1.R' once a condition is met but still continue to
 run rest of script from the 'main'R' file.
  
 A simple example is shown below. I hope only 'message1' and 'message3' can
 be shown up without using if(...) else statement for skipped part in
 'test1.R'.
  
 #main.R:
 source(test1.R)
 source(test2.R)
 ===
 #test1.R
 # In this example the goal is to skip executing the line of
 print(message2).
 # without using if() else statement.
 x - TRUE
 if (x) {
 print(message1)
 q() # want to skip rest of code in test1.R ONLY
 }
 # lots of lines can start from here.
 print(message2)
 ===
 #test2.R
 print(message3)
  
  
 Thank you.
 Ming-chung Li
 
   [[alternative HTML version deleted]]
 
 __
 R-help@r-project.org mailing list
 https://stat.ethz.ch/mailman/listinfo/r-help
 PLEASE do read the posting guide
 http://www.R-project.org/posting-guide.html
 and provide commented, minimal, self-contained, reproducible code.
 
 

-- 
View this message in context: 
http://www.nabble.com/How-to-skip-rest-of-code-tp23320816p23365295.html
Sent from the R help mailing list archive at Nabble.com.

__
R-help@r-project.org mailing list
https://stat.ethz.ch/mailman/listinfo/r-help
PLEASE do read the posting guide http://www.R-project.org/posting-guide.html
and provide commented, minimal, self-contained, reproducible code.


[R] wrong if-else syntax

2009-05-04 Thread mauede
What is wrong in the following nested if-else statements:

  if (Condition_1) {  # begin IF_1  
 statement_1
 statement_2
 statement_3
 if (Condition_2) { # begin IF_2
a- a +1
 }   # end IF_2
 statement_4  
 statement_5
 statement_6
 statement_7
 if (Condition_3) {# begin IF_2 

statement_8
 } else { # ELSE_2
statement_9
statement_10
 }  # end IF_2
  } else {   # ELSE_1
  statement_11
  }   # end IF_1


It looks like R interpret does not like the above syntax. In fact in my script 
I have the following instructions:

  if (DonohoAplhaON){
  tms - xx[,sampamp]   #EXTRACT SIGNAL 
AMPLITUDE
  tmsLen - length(tms)
  J - ilogb(tmsLen, base=2)
  if (logb(tmsLen, base=2)%%2  0) {
 J - J + 1
  }
  rm(xx)
  rawtms - tms 
  X - PreProcessor(tms,tmsLen,J)
  BestWavList - FindBestWavelet (X,tmsLen,J,Step1NumHighScalesOFF)
  if (!is.null(BestWavList)) {  
   #COMPUTE SIGNAL DONOHO-ALPHA
 features.mat[ns,alpha]- 
CalcDonohoAlpha(rawtms,BestWavList$No,J,tmsLen,Step3NumHighScalesOFF,Step3AllCoefON)
  } else {
 cat(\n\n Could not compute Best Wavelet Basis for Signal: ,fln,  
Skip current signal! \n\n)
 next
 }
  } else {
  features.mat[ns,alpha] - NA
  }

It keeps printing out an error referred to the second  } else { as if it 
could not realize that the inner if-statement has been closed:

  if(!is.null(BestWavList)) {  
#COMPUTE SIGNAL DONOHO-ALPHA
+ features.mat[ns,alpha]- 
CalcDonohoAlpha(rawtms,BestWavList$No,J,tmsLen,Step3NumHighScalesOFF,Step3AllCoefON)
+  }else {
+ cat(\n\n Could not compute Best Wavelet Basis for Signal: ,fln,  
Skip current signal! \n\n)
+ next
+  }
   }else {
Error: unexpected '}' in   }
  features.mat[ns,alpha] - NA
   }
Error: unexpected '}' in   }


Thank you for clarifying my doubts
Maura




tutti i telefonini TIM!


[[alternative HTML version deleted]]

__
R-help@r-project.org mailing list
https://stat.ethz.ch/mailman/listinfo/r-help
PLEASE do read the posting guide http://www.R-project.org/posting-guide.html
and provide commented, minimal, self-contained, reproducible code.


Re: [R] quadratic programming

2009-05-04 Thread Stephan Kolassa

Hi,

The CRAN Task View on Optimization may help:
http://stat.ethz.ch/CRAN/web/views/Optimization.html

HTH,
Stephan


barbara.r...@uniroma1.it schrieb:

Devo risolvere un problema di minimo vincolato con vincoli di uguaglianza e un 
altro con vincoli di uguaglianza e disuguaglianza.
Cosa posso utilizzare?
[[alternative HTML version deleted]]

__
R-help@r-project.org mailing list
https://stat.ethz.ch/mailman/listinfo/r-help
PLEASE do read the posting guide http://www.R-project.org/posting-guide.html
and provide commented, minimal, self-contained, reproducible code.



__
R-help@r-project.org mailing list
https://stat.ethz.ch/mailman/listinfo/r-help
PLEASE do read the posting guide http://www.R-project.org/posting-guide.html
and provide commented, minimal, self-contained, reproducible code.


[R] Odp: Need to clean a table, and compute mean and SD

2009-05-04 Thread Petr PIKAL
Hi

r-help-boun...@r-project.org napsal dne 04.05.2009 09:53:19:

 
 Hi
 
 For a school project I have a file with 120 columns and  ~2000 lines. 
This
 file contains timestamps of spike detected in 60 channels, and the time
 elapsed between the last spike.
 
 I need to clean too high values. About 98% of values are between 0 and 
2000
 and 2% are between 2000 and 20'000. I want to get rid of theses values.
 
 Please could you help me?

Decide your threshold value. Let say it is 2000. Then it depends how do 
you want to get rid of the data.

e.g.

mat[mat2000]-NA

shall put NA values instead all items which were bigger than 2000.

Regards
Petr



 
 Regards 
 
 Thom
 -- 
 View this message in context: 
http://www.nabble.com/Need-to-clean-a-table%2C-
 and-compute-mean-and-SD-tp23364460p23364460.html
 Sent from the R help mailing list archive at Nabble.com.
 
 __
 R-help@r-project.org mailing list
 https://stat.ethz.ch/mailman/listinfo/r-help
 PLEASE do read the posting guide 
http://www.R-project.org/posting-guide.html
 and provide commented, minimal, self-contained, reproducible code.

__
R-help@r-project.org mailing list
https://stat.ethz.ch/mailman/listinfo/r-help
PLEASE do read the posting guide http://www.R-project.org/posting-guide.html
and provide commented, minimal, self-contained, reproducible code.


Re: [R] odbcConnectAccess function

2009-05-04 Thread bartjoosen

You can use R CMD BATCH if I'm correctly.
How about define a query through your routine, process the results of this
query through R, write this output in a new table, or a textfile, and load
this with your subroutine.

Bart



Felipe Carrillo wrote:
 
 
 Steve: I am already using it importing data from Access to R but was
 wondering if it can be used the other way around, creating a subroutine in
 an access module and using the RODBC functions from there.
 
 
 
 --- On Thu, 4/30/09, steve_fried...@nps.gov steve_fried...@nps.gov
 wrote:
 
 From: steve_fried...@nps.gov steve_fried...@nps.gov
 Subject: Re: [R] odbcConnectAccess function
 To: mazatlanmex...@yahoo.com
 Cc: r-h...@stat.math.ethz.ch, r-help-boun...@r-project.org
 Date: Thursday, April 30, 2009, 7:38 AM
 Feilipe,
 
 You should look at the RODBC package
 
 It works great !
 
 Steve
 
 Steve Friedman Ph. D.
 Spatial Statistical Analyst
 Everglades and Dry Tortugas National Park
 950 N Krome Ave (3rd Floor)
 Homestead, Florida 33034
 
 steve_fried...@nps.gov
 Office (305) 224 - 4282
 Fax (305) 224 - 4147
 
 


  Felipe Carrillo   

  mazatlanmex...@y  
   
  ahoo.com  
To 
  Sent by: 
 r-h...@stat.math.ethz.ch
  r-help-boun...@r- 
 cc 
  project.org   


Subject 
[R]
 odbcConnectAccess function  
  04/30/2009 07:17  

  AM MST





  Please respond to 

  mazatlanmex...@ya 

   hoo.com  





 
 
 
 
 
 HI:
 Is it possible to use the RODBC package within MS Access. I
 have been using
 from R but was just wondering if it could be used along
 with R(D)COM.
 Something like this:
 
 Dim dbs As DAO.Database
 Dim rst As DAO.Recordset
 Dim myApp As StatConnector
 Set myApp = New StatConnector
 myApp.GetErrorText
 myApp.Init R
 
 myApp.EvaluateNoReturn library(RODBC)
 myApp.EvaluateNoReturn myDB -
 odbcConnectAccess(CurrentDb)
 'query the database and save it as R object 'a'
 myApp.EvaluateNoReturn a - sqlQuery(CurrentDb,
 Paste(select * From
 Sample))
 myApp.EvaluateNoReturn plot(a)
 
 Thanks in advance
 
 
 Felipe D. Carrillo
 Supervisory Fishery Biologist
 Department of the Interior
 US Fish  Wildlife Service
 California, USA
 
 __
 R-help@r-project.org mailing list
 https://stat.ethz.ch/mailman/listinfo/r-help
 PLEASE do read the posting guide
 http://www.R-project.org/posting-guide.html
 and provide commented, minimal, self-contained,
 reproducible code.
 
 __
 R-help@r-project.org mailing list
 https://stat.ethz.ch/mailman/listinfo/r-help
 PLEASE do read the posting guide
 http://www.R-project.org/posting-guide.html
 and provide commented, minimal, self-contained, reproducible code.
 
 

-- 
View this message in context: 
http://www.nabble.com/odbcConnectAccess-function-tp23317361p23365104.html
Sent from the R help mailing list archive at Nabble.com.

__
R-help@r-project.org mailing list
https://stat.ethz.ch/mailman/listinfo/r-help
PLEASE do read the posting guide http://www.R-project.org/posting-guide.html
and provide commented, minimal, self-contained, reproducible code.


[R] Randomized-complete blocks design and R

2009-05-04 Thread alis villiyam
Hi,

I am new in R.I am trying to analysis of variance for Randomized-complete
blocks design. I have 4 treatments and 3 replication,





Without intraction.then .I is going to use LSD test or Duncan test for
comparison between average values of treatments.

I want to know, is there any significant change between treatment and
between weeks? Because I did experiment for 8 weeks.



I am not familiar with R very well. Dose anybody has this Code?

Please help me.



Kind regards,

Alisa

[[alternative HTML version deleted]]

__
R-help@r-project.org mailing list
https://stat.ethz.ch/mailman/listinfo/r-help
PLEASE do read the posting guide http://www.R-project.org/posting-guide.html
and provide commented, minimal, self-contained, reproducible code.


Re: [R] clear screen?

2009-05-04 Thread Duncan Murdoch

On 03/05/2009 6:50 PM, Charles Annis, P.E. wrote:

I’ve been using this routine for several years.  I’m sorry, I don’t remember
where I got it.  It works as it should, viz. it blanks the R console.  But
it requires package rcom and now that requires rscproxy.

cls -
function () 
{

require(rcom)
wsh - comCreateObject(Wscript.Shell)
comInvoke(wsh, SendKeys, \f)
invisible(wsh)
}


cls()

Loading required package: rcom
Loading required package: rscproxy

This seems like overkill to me just to blank the R console, especially since
I am trying to diminish the number of necessary packages to support my
home-brew package.

So, is there an easier way to blank the R console in Windows?


Ctrl-L will do it.  If you want to put it in a function, I don't think so.

An item that has been on my wish list for a long time is to rewrite the 
menu system in the Windows gui so that all items can be invoked from R 
code.  It's been sitting there in the section entitled I wish someone 
else would do this.  It would not be hard, but it would be a lot of 
tedious work.


Duncan Murdoch






sessionInfo()
R version 2.8.1 (2008-12-22) 
i386-pc-mingw32 


locale:
LC_COLLATE=English_United States.1252;LC_CTYPE=English_United
States.1252;LC_MONETARY=English_United
States.1252;LC_NUMERIC=C;LC_TIME=English_United States.1252

attached base packages:
[1] splines   tcltk stats graphics  grDevices utils datasets
methods   base 


other attached packages:
[1] rcom_2.1-1 rscproxy_1.2-0 survival_2.35-3
RColorBrewer_1.0-2   RODBC_1.2-5   myhomebrew

loaded via a namespace (and not attached):
[1] tools_2.8.1



Charles Annis, P.E.

charles.an...@statisticalengineering.com
phone: 561-352-9699
eFax:  614-455-3265
http://www.StatisticalEngineering.com
 


__
R-help@r-project.org mailing list
https://stat.ethz.ch/mailman/listinfo/r-help
PLEASE do read the posting guide http://www.R-project.org/posting-guide.html
and provide commented, minimal, self-contained, reproducible code.


__
R-help@r-project.org mailing list
https://stat.ethz.ch/mailman/listinfo/r-help
PLEASE do read the posting guide http://www.R-project.org/posting-guide.html
and provide commented, minimal, self-contained, reproducible code.


Re: [R] wrong if-else syntax

2009-05-04 Thread Duncan Murdoch

On 04/05/2009 5:08 AM, mau...@alice.it wrote:

What is wrong in the following nested if-else statements:

  if (Condition_1) {  # begin IF_1  
 statement_1

 statement_2
 statement_3
 if (Condition_2) { # begin IF_2
a- a +1
 }   # end IF_2
 statement_4  
 statement_5

 statement_6
 statement_7
 if (Condition_3) {# begin IF_2 
statement_8

 } else { # ELSE_2
statement_9
statement_10
 }  # end IF_2
  } else {   # ELSE_1
  statement_11
  }   # end IF_1


It looks like R interpret does not like the above syntax. In fact in my script 
I have the following instructions:

  if (DonohoAplhaON){
  tms - xx[,sampamp]   #EXTRACT SIGNAL AMPLITUDE

  tmsLen - length(tms)
  J - ilogb(tmsLen, base=2)
  if (logb(tmsLen, base=2)%%2  0) {
 J - J + 1
  }
  rm(xx)
  rawtms - tms 
  X - PreProcessor(tms,tmsLen,J)

  BestWavList - FindBestWavelet (X,tmsLen,J,Step1NumHighScalesOFF)
  if (!is.null(BestWavList)) {  
   #COMPUTE SIGNAL DONOHO-ALPHA
 features.mat[ns,alpha]- 
CalcDonohoAlpha(rawtms,BestWavList$No,J,tmsLen,Step3NumHighScalesOFF,Step3AllCoefON)
  } else {
 cat(\n\n Could not compute Best Wavelet Basis for Signal: ,fln,  Skip 
current signal! \n\n)
 next
 }
  } else {
  features.mat[ns,alpha] - NA
  }

It keeps printing out an error referred to the second  } else { as if it 
could not realize that the inner if-statement has been closed:


I just tried the full block above, and it was fine.




 if(!is.null(BestWavList)) {
 #COMPUTE SIGNAL DONOHO-ALPHA


Notice the prompt here is , not +.  The problem occurred earlier:  R 
thinks this is a new statement, not part of the if (DonohoAplhaON) { 
block as you intended.



+ features.mat[ns,alpha]- 
CalcDonohoAlpha(rawtms,BestWavList$No,J,tmsLen,Step3NumHighScalesOFF,Step3AllCoefON)
+  }else {
+ cat(\n\n Could not compute Best Wavelet Basis for Signal: ,fln,  Skip 
current signal! \n\n)
+ next
+  }


This finishes the statement, so the else doesn't make sense.


  }else {


The spacing here doesn't match what you showed above.  Are you sure 
you're pasting the right file?


You'll get better error handling using source() than cut and paste. 
source() will stop at the first error; cut and paste just keeps going.


Duncan Murdoch



Error: unexpected '}' in   }

 features.mat[ns,alpha] - NA
  }

Error: unexpected '}' in   }


Thank you for clarifying my doubts
Maura




tutti i telefonini TIM!


[[alternative HTML version deleted]]

__
R-help@r-project.org mailing list
https://stat.ethz.ch/mailman/listinfo/r-help
PLEASE do read the posting guide http://www.R-project.org/posting-guide.html
and provide commented, minimal, self-contained, reproducible code.


__
R-help@r-project.org mailing list
https://stat.ethz.ch/mailman/listinfo/r-help
PLEASE do read the posting guide http://www.R-project.org/posting-guide.html
and provide commented, minimal, self-contained, reproducible code.


[R] Creating a variable which is the sum of equal rows in a dataframe

2009-05-04 Thread Cecilia Carmo

Hi everyone:

I need to count the number of banks of each firm in my 
data. The firm is identified by the fiscal number. The 
banks of each firm appears like this:


Firm Banks
500600700  Citybank
500600700  CGD
500600700  BES
500600800  Citybank
500600800  Bank1
500600900  CGD
…

I want to obtain the following dataframe:
Firmnumberofbanks
500600700  3
500600800  2
500600900  1
…

This is a question of counting the times each firm 
appears, but I don’t know which function do this. If 
anyone could help me I appreciate.


Thank you in advance for the help you could give me,

Cecilia Carmo (Portugal)

__
R-help@r-project.org mailing list
https://stat.ethz.ch/mailman/listinfo/r-help
PLEASE do read the posting guide http://www.R-project.org/posting-guide.html
and provide commented, minimal, self-contained, reproducible code.


Re: [R] Surface for R outside of R

2009-05-04 Thread Duncan Murdoch

On 04/05/2009 4:12 AM, koj wrote:

Hi all,

i am new here and this is my first posting. My coding experience in R ist
okay, but I have a problem which is related to another aspect of R.

I am searching for a surface which execute my commands in R via a click on a
button OUTSIDE of R. Concrete: I will write some programs in R and want to
execute this files outside of R, so I don't want to start R eyery time if i
want an analysis. The best case were a surface of e.g. 8 Buttons, each click
leads to start a specific R file. My outputs are JPEG or CSV, so I don`t
need the output inside of R. Could anyone can give me some recommendations,
what could be a solution (e. g. Java)? Is such a solution possible? 


There are many solutions.  Java (through JRI in rJava) may be the most 
portable one; there are others that are specific to particular operating 
systems.


Duncan Murdoch

__
R-help@r-project.org mailing list
https://stat.ethz.ch/mailman/listinfo/r-help
PLEASE do read the posting guide http://www.R-project.org/posting-guide.html
and provide commented, minimal, self-contained, reproducible code.


[R] Caret package: coeffcients for regression

2009-05-04 Thread Alex Roy
Dear All,
  I am using Caretpackage for SVM regression and elastic net
regression . I can get the final fiited vs observed values. How can I get
the coefficients? Any ideas?

Thanks

Alex

[[alternative HTML version deleted]]

__
R-help@r-project.org mailing list
https://stat.ethz.ch/mailman/listinfo/r-help
PLEASE do read the posting guide http://www.R-project.org/posting-guide.html
and provide commented, minimal, self-contained, reproducible code.


Re: [R] Creating a variable which is the sum of equal rows in a dataframe

2009-05-04 Thread baptiste auguie

Try this,

# d - read.table(pipe(pbpaste), head=T) # read your data

table(d)

# library(reshape)
cast(as.data.frame(table(d)), .~Firm, fun=sum)


HTH,

baptiste


On 4 May 2009, at 13:19, Cecilia Carmo wrote:


Hi everyone:

I need to count the number of banks of each firm in my
data. The firm is identified by the fiscal number. The
banks of each firm appears like this:

Firm Banks
500600700  Citybank
500600700  CGD
500600700  BES
500600800  Citybank
500600800  Bank1
500600900  CGD
…

I want to obtain the following dataframe:
Firmnumberofbanks
500600700  3
500600800  2
500600900  1
…

This is a question of counting the times each firm
appears, but I don’t know which function do this. If
anyone could help me I appreciate.

Thank you in advance for the help you could give me,

Cecilia Carmo (Portugal)

__
R-help@r-project.org mailing list
https://stat.ethz.ch/mailman/listinfo/r-help
PLEASE do read the posting guide http://www.R-project.org/posting-guide.html
and provide commented, minimal, self-contained, reproducible code.


_

Baptiste Auguié

School of Physics
University of Exeter
Stocker Road,
Exeter, Devon,
EX4 4QL, UK

Phone: +44 1392 264187

http://newton.ex.ac.uk/research/emag

__
R-help@r-project.org mailing list
https://stat.ethz.ch/mailman/listinfo/r-help
PLEASE do read the posting guide http://www.R-project.org/posting-guide.html
and provide commented, minimal, self-contained, reproducible code.


[R] levelplot question

2009-05-04 Thread Antje

Hi there,

I have a question concerning the behaviour of the colouring with levelplot. (I 
hope, I manage to explain)


If I give the parameters at and col.regions like this:

at - c(1,2,3,4,5,6)
col.regions - c(blue,blue,blue,yellow,yellow,yellow)

Which color would have the value 3.5?
I would have expected yellow, no?

In my more complex example I wanted to create a heatmap (like yellow to red) up 
to a fixed threshold. All values above should get another color (like blue).

Automatically, I generated both vectors like this:

at -c(vectorOfLowerValues, myThreshold, vectorOfHigherValues)
col.regions -  c(vectorOfHeatmapColors, lastHeatmapColor, vectorOfColorblue)

But I get some values above the threshold which are not blue... Can anybody 
explain me why? (I've checked the length of both vectors and it's parts - this 
is correct - so 'myThreshold' would get 'lastHeatmapColor' by the same vector 
position)


I'm very confused...

Antje

__
R-help@r-project.org mailing list
https://stat.ethz.ch/mailman/listinfo/r-help
PLEASE do read the posting guide http://www.R-project.org/posting-guide.html
and provide commented, minimal, self-contained, reproducible code.


[R] Odp: Creating a variable which is the sum of equal rows in a dataframe

2009-05-04 Thread Petr PIKAL
Hi
r-help-boun...@r-project.org napsal dne 04.05.2009 13:19:15:

 Hi everyone:
 
 I need to count the number of banks of each firm in my 
 data. The firm is identified by the fiscal number. The 
 banks of each firm appears like this:
 
 Firm Banks
 500600700  Citybank
 500600700  CGD
 500600700  BES
 500600800  Citybank
 500600800  Bank1
 500600900  CGD
 …
 
 I want to obtain the following dataframe:
 Firmnumberofbanks
 500600700  3
 500600800  2
 500600900  1
 …

E.g.

as.data.frame(rowSums(table(test)))

Regards
Petr

 
 This is a question of counting the times each firm 
 appears, but I don’t know which function do this. If 
 anyone could help me I appreciate.
 
 Thank you in advance for the help you could give me,
 
 Cecilia Carmo (Portugal)
 
 __
 R-help@r-project.org mailing list
 https://stat.ethz.ch/mailman/listinfo/r-help
 PLEASE do read the posting guide 
http://www.R-project.org/posting-guide.html
 and provide commented, minimal, self-contained, reproducible code.

__
R-help@r-project.org mailing list
https://stat.ethz.ch/mailman/listinfo/r-help
PLEASE do read the posting guide http://www.R-project.org/posting-guide.html
and provide commented, minimal, self-contained, reproducible code.


Re: [R] Creating a variable which is the sum of equal rows in a dataframe

2009-05-04 Thread Fredrik Karlsson
Hi Cecilia,

You can use table for this:

#Generate a dataframe
 dat.df - data.frame(Firm = 
 sample(c(500600700,500600800,500600800),6,replace=TRUE),Banks=sample(c(Citybank,CGD,DES,Bank1),6,replace=TRUE)
   )

#Get the counts in a table format
 with(dat.df, table(Firm,Banks))
   Banks
FirmBank1 CGD Citybank DES
  500600700 0   01   0
  500600800 2   11   1

#Or, if you prefer the data.frame format, you can reshape it.
 as.data.frame(with(dat.df, table(Firm,Banks)))
   FirmBanks Freq
1 500600700Bank10
2 500600800Bank12
3 500600700  CGD0
4 500600800  CGD1
5 500600700 Citybank1
6 500600800 Citybank1
7 500600700  DES0
8 500600800  DES1

Hope this helps.

/Fredrik

On Mon, May 4, 2009 at 1:19 PM, Cecilia Carmo cecilia.ca...@ua.pt wrote:
 Hi everyone:

 I need to count the number of banks of each firm in my data. The firm is
 identified by the fiscal number. The banks of each firm appears like this:

 Firm                     Banks
 500600700          Citybank
 500600700          CGD
 500600700          BES
 500600800          Citybank
 500600800          Bank1
 500600900          CGD
 …

 I want to obtain the following dataframe:
 Firm            numberofbanks
 500600700          3
 500600800          2
 500600900          1
 …

 This is a question of counting the times each firm appears, but I don’t know
 which function do this. If anyone could help me I appreciate.

 Thank you in advance for the help you could give me,

 Cecilia Carmo (Portugal)

 __
 R-help@r-project.org mailing list
 https://stat.ethz.ch/mailman/listinfo/r-help
 PLEASE do read the posting guide http://www.R-project.org/posting-guide.html
 and provide commented, minimal, self-contained, reproducible code.




-- 
Life is like a trumpet - if you don't put anything into it, you don't
get anything out of it.

__
R-help@r-project.org mailing list
https://stat.ethz.ch/mailman/listinfo/r-help
PLEASE do read the posting guide http://www.R-project.org/posting-guide.html
and provide commented, minimal, self-contained, reproducible code.


Re: [R] clear screen?

2009-05-04 Thread Gabor Grothendieck
On Mon, May 4, 2009 at 6:50 AM, Duncan Murdoch murd...@stats.uwo.ca wrote:
 On 03/05/2009 6:50 PM, Charles Annis, P.E. wrote:

 I’ve been using this routine for several years.  I’m sorry, I don’t
 remember
 where I got it.  It works as it should, viz. it blanks the R console.  But
 it requires package rcom and now that requires rscproxy.

 cls -
 function () {
    require(rcom)
    wsh - comCreateObject(Wscript.Shell)
    comInvoke(wsh, SendKeys, \f)
    invisible(wsh)
 }

 cls()

 Loading required package: rcom
 Loading required package: rscproxy

 This seems like overkill to me just to blank the R console, especially
 since
 I am trying to diminish the number of necessary packages to support my
 home-brew package.

 So, is there an easier way to blank the R console in Windows?

 Ctrl-L will do it.  If you want to put it in a function, I don't think so.

 An item that has been on my wish list for a long time is to rewrite the menu
 system in the Windows gui so that all items can be invoked from R code.
  It's been sitting there in the section entitled I wish someone else would
 do this.  It would not be hard, but it would be a lot of tedious work.

Note that the same technique used to clear the screen could be used for
this as well. For example, this causes the FAQ to appear by sending
the appropriate set of characters to active the menu:

sendKeys - function (x) {
require(RDCOMClient)
wsh - COMCreate(Wscript.Shell)
wsh$SendKeys(x)
invisible(wsh)
}
faq - function() sendKeys(%HF\n)

faq()

Might have to add some Sys.sleep calls to slow it down but so far it
seems to work for me.

__
R-help@r-project.org mailing list
https://stat.ethz.ch/mailman/listinfo/r-help
PLEASE do read the posting guide http://www.R-project.org/posting-guide.html
and provide commented, minimal, self-contained, reproducible code.


Re: [R] Creating a variable which is the sum of equal rows in a dataframe

2009-05-04 Thread Gabor Grothendieck
Try aggregate. First we read the data into DF and
then apply aggregate:

 Lines - Firm Banks
+ 500600700  Citybank
+ 500600700  CGD
+ 500600700  BES
+ 500600800  Citybank
+ 500600800  Bank1
+ 500600900  CGD
 # DF - read.table(myfile.dat, header = TRUE)
 DF - read.table(textConnection(Lines), header = TRUE)

 aggregate(DF[Banks], DF[Firm], length)
   Firm Banks
1 500600700 3
2 500600800 2
3 500600900 1

On Mon, May 4, 2009 at 7:19 AM, Cecilia Carmo cecilia.ca...@ua.pt wrote:
 Hi everyone:

 I need to count the number of banks of each firm in my data. The firm is
 identified by the fiscal number. The banks of each firm appears like this:

 Firm                     Banks
 500600700          Citybank
 500600700          CGD
 500600700          BES
 500600800          Citybank
 500600800          Bank1
 500600900          CGD
 …

 I want to obtain the following dataframe:
 Firm            numberofbanks
 500600700          3
 500600800          2
 500600900          1
 …

 This is a question of counting the times each firm appears, but I don’t know
 which function do this. If anyone could help me I appreciate.

 Thank you in advance for the help you could give me,

 Cecilia Carmo (Portugal)

 __
 R-help@r-project.org mailing list
 https://stat.ethz.ch/mailman/listinfo/r-help
 PLEASE do read the posting guide http://www.R-project.org/posting-guide.html
 and provide commented, minimal, self-contained, reproducible code.


__
R-help@r-project.org mailing list
https://stat.ethz.ch/mailman/listinfo/r-help
PLEASE do read the posting guide http://www.R-project.org/posting-guide.html
and provide commented, minimal, self-contained, reproducible code.


Re: [R] About the Transfer Function Model(ARIMAX)

2009-05-04 Thread David Winsemius

pages 43-54 of:

http://cran.r-project.org/doc/Rnews/Rnews_2006-4.pdf

... is an article by Uwe Ligges on accesssing source code.

On Apr 2, 2009, at 4:20 AM, Ramanath Roy wrote:


Dear ALL,

I would appreciate if someone help me by letting me know the code of  
above model in R.I would request you to please let me know how i could

make arimax model in auto.arima.




David Winsemius, MD
Heritage Laboratories
West Hartford, CT

__
R-help@r-project.org mailing list
https://stat.ethz.ch/mailman/listinfo/r-help
PLEASE do read the posting guide http://www.R-project.org/posting-guide.html
and provide commented, minimal, self-contained, reproducible code.


[R] how to remove ( ) ? when using write.csv(chron.object)

2009-05-04 Thread Qianfeng Li
Hello all,

I don' like to have ( ) around my datetime, when send the csv file to somebody. 

How to remove it?

library(chron)
mydate - c(2009-05-03,2009-06-07)
mytime - c(10:30:00,10:20:00)
mydatetime - chron(dates=mydate,times=mytime,format=c(y-m-d,h:m:s))
write.csv(mydatetime)
 
,x
1,(09-05-03 10:30:00)
2,(09-06-07 10:20:00)
 
thanks!
Jeff 


  
[[alternative HTML version deleted]]

__
R-help@r-project.org mailing list
https://stat.ethz.ch/mailman/listinfo/r-help
PLEASE do read the posting guide http://www.R-project.org/posting-guide.html
and provide commented, minimal, self-contained, reproducible code.


[R] levelplot question

2009-05-04 Thread Antje

@Dieter:

 You implicitly expect round().

 Your question implies that you may also susceptible to the problem of
 R FAQ 7.31, Why doesn't R think these numbers are equal?

No, I guess, you misunderstood my question. These vectors (at and col.regions) 
are given to levelplot together with some data which should be visualized.
I don't know how exactly levelplot works (I was asking for help to understand 
it) but somehow it creates something like bins from at and uses col.regions 
to display all datavalues within the same bin with the according color.
I'd like to know, which color to expect from levelplot for a certain datavalue 
(3.5)


Did I explain it somehow clear now?

Antje

__
R-help@r-project.org mailing list
https://stat.ethz.ch/mailman/listinfo/r-help
PLEASE do read the posting guide http://www.R-project.org/posting-guide.html
and provide commented, minimal, self-contained, reproducible code.


Re: [R] Surface for R outside of R

2009-05-04 Thread Hans-Peter Suter
 want an analysis. The best case were a surface of e.g. 8 Buttons, each click
 leads to start a specific R file. My outputs are JPEG or CSV, so I don`t
 need the output inside of R. Could anyone can give me some recommendations,
 what could be a solution (e. g. Java)? Is such a solution possible?

 What about batch scripts which would be called by your GUI? (maybe
you could even skip the gui...)
(see ?Rscript (unix alike) or Windows FAQ, 2.12)

--
Regards,
Hans-Peter

__
R-help@r-project.org mailing list
https://stat.ethz.ch/mailman/listinfo/r-help
PLEASE do read the posting guide http://www.R-project.org/posting-guide.html
and provide commented, minimal, self-contained, reproducible code.


Re: [R] how to remove ( ) ? when using write.csv(chron.object)

2009-05-04 Thread Gabor Grothendieck
Try:

 write.csv(format(mydatetime, enclosed = c(, )))
,x
1,09-05-03 10:30:00
2,09-06-07 10:20:00


On Mon, May 4, 2009 at 10:34 AM, Qianfeng Li qflic...@yahoo.com wrote:
 Hello all,

 I don' like to have ( ) around my datetime, when send the csv file to 
 somebody.

 How to remove it?

library(chron)
mydate - c(2009-05-03,2009-06-07)
mytime - c(10:30:00,10:20:00)
mydatetime - chron(dates=mydate,times=mytime,format=c(y-m-d,h:m:s))
write.csv(mydatetime)

 ,x
 1,(09-05-03 10:30:00)
 2,(09-06-07 10:20:00)

 thanks!
 Jeff



        [[alternative HTML version deleted]]


 __
 R-help@r-project.org mailing list
 https://stat.ethz.ch/mailman/listinfo/r-help
 PLEASE do read the posting guide http://www.R-project.org/posting-guide.html
 and provide commented, minimal, self-contained, reproducible code.



__
R-help@r-project.org mailing list
https://stat.ethz.ch/mailman/listinfo/r-help
PLEASE do read the posting guide http://www.R-project.org/posting-guide.html
and provide commented, minimal, self-contained, reproducible code.


Re: [R] Support Vector Machines

2009-05-04 Thread David Winsemius


On May 4, 2009, at 8:52 AM, excalibur wrote:



This question is still unanswered.


Unanswered questions are often those which do not comply with the  
guidelines in the Posting Guide. Many people have gotten tired of  
either making up examples or of writing  Read the Posting Guide, so  
they just ignore them.



Someone can explain me how use the svm function to make density  
estimation ?


If you post an executable bit of code that shows how you are doing  
those operations, then I suspect someone will answer.


excalibur wrote:




Hi,
i try to use function svm of package e1071 to estimate a density.

But if my data are X=(X1,...,Xn) and m-svm(X) some values of m$SV  
are

less than 0.
I don't see how i can get the estimation of the density with this
function.

Thanks for your help.

Rémi



David Winsemius, MD
Heritage Laboratories
West Hartford, CT

__
R-help@r-project.org mailing list
https://stat.ethz.ch/mailman/listinfo/r-help
PLEASE do read the posting guide http://www.R-project.org/posting-guide.html
and provide commented, minimal, self-contained, reproducible code.


Re: [R] Caret package: coeffcients for regression

2009-05-04 Thread Max Kuhn
Alex,

 I am using Caretpackage for SVM regression and elastic net
 regression . I can get the final fiited vs observed values. How can I get
 the coefficients? Any ideas?

You didn't say what version of caret and R you are using, what kernel
or what type of coefficients.

If you tune a model using train:

  fit - train(x, y, enet)

then fit$finalModel contains an enet object with the final parameters.
You can use coef() or any other appropriate function on this object.

In the case of the elastic net, see ?predict.enet, specifically the
type argument to get the regression coefficients.

If you need to get to the final tuning parameters for the model, you
can access them via x$finalModel$tuneValue. There are a few exceptions
to this: models using S4 classes won't allow adding new objects.

For SVMs, you haven't said what kernel type you used. caret uses
kernel methods from kernlab (for no other reason that I like the
package). The only package that I know gives the *regression*
coefficients for linear kernels is svmpath (there could be others). If
you want the SV coefficients (typically denoted by alpha), kernlab has
a slot called coef that you can use.

You should really provide more information for your questions to be
answered well.

-- 

Max

__
R-help@r-project.org mailing list
https://stat.ethz.ch/mailman/listinfo/r-help
PLEASE do read the posting guide http://www.R-project.org/posting-guide.html
and provide commented, minimal, self-contained, reproducible code.


[R] Reversing axis label order

2009-05-04 Thread Steve Murray

Dear R Users,

I am executing the following command to produce a line graph:

matplot(aggregate_1986[,1], aggregate_1986[,2:3], type=l, col=2:3)

On the x-axis I have values of Latitude (in column 1) ranging from -60 to +80 
(left to right on the x-axis). However, I wish to have these values shown in 
reverse on the x-axis, going from +80 to -60 (ie. North to South in terms of 
Latitude). I have tried doing this by altering the command as follows:

matplot(-aggregate_1986[,1], aggregate_1986[,2:3], type=l, col=2:3)

...but this produces the inverse sign of the latitude values along the axis - 
ie. it goes from -80 to +60.

How do I reverse the display of the axis labels correctly and of course, 
maintain the associated data values correctly?

Many thanks,

Steve

__
R-help@r-project.org mailing list
https://stat.ethz.ch/mailman/listinfo/r-help
PLEASE do read the posting guide http://www.R-project.org/posting-guide.html
and provide commented, minimal, self-contained, reproducible code.


[R] mosaic plot with two factors and one continous variable

2009-05-04 Thread Karsten Weinert
Hello,
my dataset is already aggregated: I have two categorical variables, say
continent and country and one non-negative metric variable population.
Now I would like to produce a mosaic plot which uses population to
determine how large each area of the plot is.

In other words, I fail to find out
- how to use the mosaic function of the vcd package without using table,
- or I do not know how to convert my dataset into a table (as.table fails)

As an example, here a simple dataset:

Continent; Country; Population
Europe; France; 65
Europe; Germany; 80
Africa; Algeria; 32


Can you help me to get started?

Kind regards,
Karsten.

[[alternative HTML version deleted]]

__
R-help@r-project.org mailing list
https://stat.ethz.ch/mailman/listinfo/r-help
PLEASE do read the posting guide http://www.R-project.org/posting-guide.html
and provide commented, minimal, self-contained, reproducible code.


[R] can install.packages() copy utility files to the public_html directory ?

2009-05-04 Thread Markus Loecher
Dear fellow R-users,
I am about to publish an HTML utility package to CRAN that expands on the
R2HTML package and includes a few goodies such as sorted tables, easy
automation of framed HTML reporting, etc.
However, some of the resulting dynamic HTML pages need to access JavaScript
code that should sit in a specific subdirectory of public_html.
My more general question is hence, (i) how do I include the directory
containing the JavaScript code in my R package and (ii) is it possible to
copy this directory to the user's public_html path during installation ?

Thanks!
Markus

[[alternative HTML version deleted]]

__
R-help@r-project.org mailing list
https://stat.ethz.ch/mailman/listinfo/r-help
PLEASE do read the posting guide http://www.R-project.org/posting-guide.html
and provide commented, minimal, self-contained, reproducible code.


Re: [R] Dynamic visualisation of R data using Adobe FLEX

2009-05-04 Thread Harsh
Hi Yihui,
In trying to make FLEX and R talk, I have decided to approach it by
using the Rserver JRclient as the middle tier in a three tier
approach.
Tier 1: FLEX Visualisation
Tier 2: JRclient (http://rosuda.org/Rserve/example.shtml)
Tier 3: Rserver

I have yet to begin working on this front, but would like to know what
you think about this approach. I understand FLEX and JAVA communicate
seamlessly and a JAVA client to communicate with Rserver seems to seal
the deal for making FLEX and R talk to each other.

Thanks
Harsh Singhal




On Sat, May 2, 2009 at 12:37 PM, Yihui Xie xieyi...@gmail.com wrote:
 Hi Harsh,

 If your project is so specific on the regression model, my suggestion
 is to use tcl/tk or Gtk2 interface instead of FLEX, because your
 requirement on interaction is simple (e.g. you don't need to
 drag-and-drop points).

 I think the example 'run.cor2.examp()' in 'TeachingDemos' package can
 give you enough inspiration.

 Or if you want to do the job (to change the slope) automatically, you
 may want to see the example in the 'animation' package:

 ##
 install.packages(animation)
 library(animation)

 # default animation in R: with slope changing
 least.squares()

 # animation in an HTML page
 oopt = ani.options(ani.height = 450, ani.width = 600,
    outdir = tempdir(), nmax = 50, title = Demonstration of Least Squares,
    description = We want to find an estimate for the slope
         in 50 candidate slopes, so we just compute the RSS one by one. )
 ani.start()
 par(mar = c(4, 4, 0.5, 0.1), mgp = c(2, 0.5, 0), tcl = -0.3)
 least.squares()
 ani.stop()
 ani.options(oopt)
 ##

 P. S. I'm quite interested in interactive visualization using R and
 Flash (you may also want to know Flare and SWF Tools!), so please let
 me know if there's any progress in your project.

 Regards,
 Yihui
 --
 Yihui Xie xieyi...@gmail.com
 Phone: +86-(0)10-82509086 Fax: +86-(0)10-82509086
 Mobile: +86-15810805877
 Homepage: http://www.yihui.name
 School of Statistics, Room 1037, Mingde Main Building,
 Renmin University of China, Beijing, 100872, China



 On Thu, Apr 30, 2009 at 2:54 AM, Harsh singhal...@gmail.com wrote:
 Hi useRs,

 I had posted about Adobe FLEX talking to R for rich visualisation.
 Reply from Jeffery Horner contained links to the
 revolution-computing.com webpage which had information pertaining to
 the Bay Users R group Meetup on Web Dashboards with R.

 I have a very specific project that I need to implement.
 I wish to use the graphics capabilities provided by Adobe FLEX to
 visualise outputs from R.
 For example:
 I would like to fit a regression model to a dataset in R and provide a
 FLEX interface wherein, a user may manipulate a slider which would
 increase or decrease
 a parameter estimate (between the confidence intervals of the
 estimate) of a variable to see the effect on the predicted values.

 I necessarily have to use FLEX for the interface.
 In trying to make sense of how to go about this task, I have
 speculated on the following technologies:
 1) Using RSOAP which requires a Python client. I am not sure how I can
 implement this in Flex.
 2) Using RSOAP provided by the Biocep project. This is a java
 implementation and maybe this would suit my requirements.
 3) The StatDataML package in R provides the creation of xml files of R
 objects. Maybe I can use these XML files as encapsulators of R object
 data and pass it on to FLEX.
 4) Since R has interfaces that connect to most SQL database servers, I
 can convert results of lm objects (summary(lmobject)) to data frames
 and push them into the SQL database. FLEX can
 then read the table and extract the relevant fields for output.

 I am aware that I not a computer programmer which is why I may not be
 making a whole lot of sense in describing the above scenarios, but I
 am trying to gain as much information and programming skills to
 'decipher'
 the secret behind using FLEX and R.

 I have looked at Jeffery Horner's  BBPLOT project
 (http://data.vanderbilt.edu/rapache/bbplot) and honestly I'm amazed.
 Replicating this with a FLEX interface is the closest I can get in
 describing my requirements.

 Any information/technical sources/tutorials etc in this regard will be
 much appreciated.

 Thank you for your time.

 Regards
 Harsh Singhal
 Decision Systems
 Mu Sigma Inc.
 Chicago, IL



__
R-help@r-project.org mailing list
https://stat.ethz.ch/mailman/listinfo/r-help
PLEASE do read the posting guide http://www.R-project.org/posting-guide.html
and provide commented, minimal, self-contained, reproducible code.


Re: [R] can install.packages() copy utility files to the public_html directory ?

2009-05-04 Thread Gabor Grothendieck
2009/5/4 Uwe Ligges lig...@statistik.tu-dortmund.de:


 Markus Loecher wrote:

 Dear fellow R-users,
 I am about to publish an HTML utility package to CRAN that expands on the
 R2HTML package and includes a few goodies such as sorted tables, easy
 automation of framed HTML reporting, etc.
 However, some of the resulting dynamic HTML pages need to access
 JavaScript
 code that should sit in a specific subdirectory of public_html.
 My more general question is hence, (i) how do I include the directory
 containing the JavaScript code in my R package and

 Yes, in directory ./inst/JavaScript for example that is copied to
 ./JavaScript during installation of the package.


 (ii) is it possible to
 copy this directory to the user's public_html path during installation ?

 Yes, if it is a source package (not for a binary package), BUT you should
 not write such code that copyies files to a directory different than the
 package's directory in the library that it is installed to. Things that
 install files to suprising locations are sometimes considered to be some
 kind of malware.


Also not all uses of R2HTML would be for use with a web
server at all.  Also you can't assume the directory is public_html
even if it is on a server.

__
R-help@r-project.org mailing list
https://stat.ethz.ch/mailman/listinfo/r-help
PLEASE do read the posting guide http://www.R-project.org/posting-guide.html
and provide commented, minimal, self-contained, reproducible code.


Re: [R] normality test for large a large dataset ?

2009-05-04 Thread Ben Bolker



Martial Sankar wrote:
 
 Do you know a R implemented normality test like the shapiro test but more
 suitable for large data set ?
 

Try installing the nortest package.

 
-- 
View this message in context: 
http://www.nabble.com/normality-test-for-large-a-large-dataset---tp23364526p23366862.html
Sent from the R help mailing list archive at Nabble.com.

__
R-help@r-project.org mailing list
https://stat.ethz.ch/mailman/listinfo/r-help
PLEASE do read the posting guide http://www.R-project.org/posting-guide.html
and provide commented, minimal, self-contained, reproducible code.


Re: [R] levelplot question

2009-05-04 Thread Dieter Menne
Antje niederlein-rstat at yahoo.de writes:

 I have a question concerning the behaviour of the colouring with levelplot.  
 If I give the parameters at and col.regions like this:
 
 at - c(1,2,3,4,5,6)
 col.regions - c(blue,blue,blue,yellow,yellow,yellow)
 
 Which color would have the value 3.5?
 I would have expected yellow, no?

You implicitly expect round(). 

[1] blue
 col.regions[3.5]
[1] blue
 col.regions[round(3.5)]
[1] yellow

Your question implies that you may also susceptible to the problem of
R FAQ 7.31, Why doesn't R think these numbers are equal?

Dieter

__
R-help@r-project.org mailing list
https://stat.ethz.ch/mailman/listinfo/r-help
PLEASE do read the posting guide http://www.R-project.org/posting-guide.html
and provide commented, minimal, self-contained, reproducible code.


Re: [R] Need to clean a table, and compute mean and SD

2009-05-04 Thread Luc Villandre

Thom_249 wrote:

Hi

For a school project I have a file with 120 columns and  ~2000 lines. This
file contains timestamps of spike detected in 60 channels, and the time
elapsed between the last spike.

I need to clean too high values. About 98% of values are between 0 and 2000
and 2% are between 2000 and 20'000. I want to get rid of theses values.

Please could you help me?

Regards 


Thom
  

Hi,

If you are dealing with a data frame, you can simply use the index 
function like this,


my.data.frame[my.data.frame2000] = NA

This way, all values in your data.frame that are greater than 2000 will 
be transformed into NAs. However, this kind of substitution wouldn't be 
recommended if your goal is to fit a regression model to the data. 
Missing data can sometimes be informative, hence the importance to be 
careful when doing such substitutions.

--
*Luc Villandré*

__
R-help@r-project.org mailing list
https://stat.ethz.ch/mailman/listinfo/r-help
PLEASE do read the posting guide http://www.R-project.org/posting-guide.html
and provide commented, minimal, self-contained, reproducible code.


Re: [R] Reversing axis label order

2009-05-04 Thread Patrizio Frederic
Dear Steve,
I'm not sure I properly understood the question. Try

x- aggregate_1986 # just a shortcut
n- dim(x)[[1]]
matplot(x[n:1,1], x[n:1,2:3], type=l, col=2:3)

is that what you needed?

Patrizio

2009/5/4 Steve Murray smurray...@hotmail.com:

 Dear R Users,

 I am executing the following command to produce a line graph:

 matplot(aggregate_1986[,1], aggregate_1986[,2:3], type=l, col=2:3)

 On the x-axis I have values of Latitude (in column 1) ranging from -60 to +80 
 (left to right on the x-axis). However, I wish to have these values shown in 
 reverse on the x-axis, going from +80 to -60 (ie. North to South in terms of 
 Latitude). I have tried doing this by altering the command as follows:

 matplot(-aggregate_1986[,1], aggregate_1986[,2:3], type=l, col=2:3)

 ...but this produces the inverse sign of the latitude values along the axis - 
 ie. it goes from -80 to +60.

 How do I reverse the display of the axis labels correctly and of course, 
 maintain the associated data values correctly?

 Many thanks,

 Steve

 __
 R-help@r-project.org mailing list
 https://stat.ethz.ch/mailman/listinfo/r-help
 PLEASE do read the posting guide http://www.R-project.org/posting-guide.html
 and provide commented, minimal, self-contained, reproducible code.




-- 
+-
| Patrizio Frederic, PhD
| Assistant Professor,
| Department of Economics,
| University of Modena and Reggio Emilia,
| Via Berengario 51,
| 41100 Modena, Italy
|
| tel:  +39 059 205 6727
| fax:  +39 059 205 6947
| mail: patrizio.frede...@unimore.it
+-

__
R-help@r-project.org mailing list
https://stat.ethz.ch/mailman/listinfo/r-help
PLEASE do read the posting guide http://www.R-project.org/posting-guide.html
and provide commented, minimal, self-contained, reproducible code.


Re: [R] mosaic plot with two factors and one continous variable

2009-05-04 Thread Achim Zeileis



On Mon, 4 May 2009, Karsten Weinert wrote:


Hello,
my dataset is already aggregated: I have two categorical variables, say
continent and country and one non-negative metric variable population.
Now I would like to produce a mosaic plot which uses population to
determine how large each area of the plot is.

In other words, I fail to find out
- how to use the mosaic function of the vcd package without using table,
- or I do not know how to convert my dataset into a table (as.table fails)

As an example, here a simple dataset:

Continent; Country; Population
Europe; France; 65
Europe; Germany; 80
Africa; Algeria; 32


Can you help me to get started?


You can create the tables by
  xtabs(Population ~ Continent, data = mydata)
etc. which can then be visualized using mosaic() in the usual way.

hth,
Z


Kind regards,
Karsten.

[[alternative HTML version deleted]]

__
R-help@r-project.org mailing list
https://stat.ethz.ch/mailman/listinfo/r-help
PLEASE do read the posting guide http://www.R-project.org/posting-guide.html
and provide commented, minimal, self-contained, reproducible code.




__
R-help@r-project.org mailing list
https://stat.ethz.ch/mailman/listinfo/r-help
PLEASE do read the posting guide http://www.R-project.org/posting-guide.html
and provide commented, minimal, self-contained, reproducible code.


Re: [R] Support Vector Machines

2009-05-04 Thread excalibur

This question is still unanswered.

Someone can explain me how use the svm function to make density estimation ? 

Thanks ...



excalibur wrote:
 
 Hi,
 i try to use function svm of package e1071 to estimate a density.
 
 But if my data are X=(X1,...,Xn) and m-svm(X) some values of m$SV are
 less than 0.
 I don't see how i can get the estimation of the density with this
 function.
 
 Thanks for your help.
 
 Rémi
 

-- 
View this message in context: 
http://www.nabble.com/Support-Vector-Machines-tp19069442p23366761.html
Sent from the R help mailing list archive at Nabble.com.

__
R-help@r-project.org mailing list
https://stat.ethz.ch/mailman/listinfo/r-help
PLEASE do read the posting guide http://www.R-project.org/posting-guide.html
and provide commented, minimal, self-contained, reproducible code.


[R] Zelig, oprobit error

2009-05-04 Thread Jennifer Brea

Hello,

I'm getting an error message when I use the ordered probit model 
oprobit in the zelig function.  Using the same form as in the help 
file, we get an error message.  It produces coefficients, but no 
standard errors.  See results below.  Any hints?


Thanks!

 
 o.probit - zelig(as.factor(checks.change) ~ Oda + Oil + sh_neg + 
sh_pos + checks1, model=oprobit, data=data.all)

 summary(o.probit)
Error in svd(X) : infinite or missing values in 'x'
 o.probit$coefficients
  OdaOil sh_neg sh_poschecks1 
2.858e-02  1.978e-07 -1.961e-02  4.810e-02  8.492e-02 



__
R-help@r-project.org mailing list
https://stat.ethz.ch/mailman/listinfo/r-help
PLEASE do read the posting guide http://www.R-project.org/posting-guide.html
and provide commented, minimal, self-contained, reproducible code.


[R] Handling Error Thrown After source(...), read.table(...), and readLines(...)

2009-05-04 Thread Jason Rupert

Finally, I'm getting to a point in working with R where I would like to start 
to add some smart and custom error handling.  

Specifically, I would like to add custom error handling to my code for 
source(...), read.table(...), and readLines(...)

For example, right now I have the following:
 postprocesspath-c(C:/)
 postprocess_scriptname-c(Bob.R)
 fullpostpocessingpath-paste(postprocesspath, postprocess_scriptname, sep=)
 source(fullpostpocessingpath)
Error in file(file, r, encoding = encoding) : 
  cannot open the connection
In addition: Warning message:
In file(file, r, encoding = encoding) :
  cannot open file 'C:/Bob.R': No such file or directory


Instead of the error shown above, I would just like to simply have the last 
line cannot open file 'C:/Bob.R': No such file or directory Or, add my own 
statement to the above, e.g. 

##
##
[[elided Yahoo spam]]
##
##


I would like to do something similar for read.table(...) and readLines(...).


Is such a thing possible? 

Thanks again.

__
R-help@r-project.org mailing list
https://stat.ethz.ch/mailman/listinfo/r-help
PLEASE do read the posting guide http://www.R-project.org/posting-guide.html
and provide commented, minimal, self-contained, reproducible code.


Re: [R] normality test for large a large dataset ?

2009-05-04 Thread Greg Snow
See: http://finzi.psych.upenn.edu/R/Rhelp08/archive/136160.html

But the issue really comes down to the fact that the questions: exactly 
normal?, and normal enough? are 2 very different questions (with the 
difference becoming greater with increased sample size) and while the first is 
the easier to answer, the second is generally the more useful one.

Hope this helps,

-- 
Gregory (Greg) L. Snow Ph.D.
Statistical Data Center
Intermountain Healthcare
greg.s...@imail.org
801.408.8111


 -Original Message-
 From: r-help-boun...@r-project.org [mailto:r-help-boun...@r-
 project.org] On Behalf Of Martial Sankar
 Sent: Monday, May 04, 2009 1:49 AM
 To: rhelp
 Subject: [R] normality test for large a large dataset ?
 
 
 
 Hello,
 
 Do you know a R implemented normality test like the shapiro test but
 more suitable for large data set ?
 
 Thanks,
 
 
 
 _
 Découvrez toutes les possibilités de communication avec vos proches
 
   [[alternative HTML version deleted]]

__
R-help@r-project.org mailing list
https://stat.ethz.ch/mailman/listinfo/r-help
PLEASE do read the posting guide http://www.R-project.org/posting-guide.html
and provide commented, minimal, self-contained, reproducible code.


[R] Cox Proportional Odds Model

2009-05-04 Thread Steve Jaffe

Is there a function, like coxph for the proportional hazard model, for
fitting a discrete-time proportional odds model? 
-- 
View this message in context: 
http://www.nabble.com/Cox-Proportional-Odds-Model-tp23373426p23373426.html
Sent from the R help mailing list archive at Nabble.com.

__
R-help@r-project.org mailing list
https://stat.ethz.ch/mailman/listinfo/r-help
PLEASE do read the posting guide http://www.R-project.org/posting-guide.html
and provide commented, minimal, self-contained, reproducible code.


[R] Code of the max() function

2009-05-04 Thread Paul Smith
Dear All,

Where can one find the code of the max() function? Is that written in C?

Thanks in advance,

Paul

__
R-help@r-project.org mailing list
https://stat.ethz.ch/mailman/listinfo/r-help
PLEASE do read the posting guide http://www.R-project.org/posting-guide.html
and provide commented, minimal, self-contained, reproducible code.


[R] help_if command_new column

2009-05-04 Thread barbara horta e costa
Hi everyone,

Do you know how to apply a If command to a vector in a data.frame, creating
a new column?

Example: If one species has 1 value in the Abundance column then it is
Solitary. If the abundance is higher than 1, is School.

I did:

aggregationFunction -function(x){
 (if (x1) a- 'School'
else a- 'Solitary')
return (a)
   }

 Then:

Aggregation - lapply (df$Number, aggregationFunction )



Then:

 df - data.frame (df, Aggregation)



And I got this error:
Error in data.frame(df, Aggregation) :
  arguments imply differing number of rows: 2576, 1



Can you help me doing this function to a vector? I’ve tried to find this
question and the answer online but I couldn't.



I tried also only with if but it is only for 1 length and the error was:

  the condition has length  1 and only the first element will be used



This question seemed easy in the beginning but I was not successful.



Thanks a lot for helping me.

Barbara


-- 
Bárbara H. Costa
Marine Biologist Researcher
SCIAENA - Marine Sciences and Cooperation
www.sciaena.org

bco...@ispa.pt
barbarahco...@gmail.com

[[alternative HTML version deleted]]

__
R-help@r-project.org mailing list
https://stat.ethz.ch/mailman/listinfo/r-help
PLEASE do read the posting guide http://www.R-project.org/posting-guide.html
and provide commented, minimal, self-contained, reproducible code.


[R] Java-R

2009-05-04 Thread andrea.sp...@quantide.com

Dear R user
We would be interested to talk with someone, living in Milan (Italy) 
area, with good experience in Java-R based applications development.

We can provide good R skills but we are poor java programmers
Can anyone help?
Thanks in advance for any help


--
Andrea Spano'
Quantide s.r.l.

andrea.sp...@quantide.com

Il messaggio trasmesso può contenere informazioni di carattere
confidenziale rivolte esclusivamente al destinatario. Ne è vietato
l'uso, l diffusione, la distribuzione o la riproduzione da parte di
altre persone e/o entità diverse da quelle specificate. Nel caso aveste
ricevuto questo messaggio per errore siete pregati di segnalarlo
immediatamente al mittente e cancellare quanto ricevuto.

This electronic mail transmission may contain confidential information
addressed only to the person (s) named. Any use, distribution, copyng or
disclosure by any other person and/or entities other than the intended
recipient is prohibited. If you received this transmission in error,
please inform the sender immediately and delete the material.

__
R-help@r-project.org mailing list
https://stat.ethz.ch/mailman/listinfo/r-help
PLEASE do read the posting guide http://www.R-project.org/posting-guide.html
and provide commented, minimal, self-contained, reproducible code.


Re: [R] Hierarchical Diagram of Networks in sna or otherwise?

2009-05-04 Thread jebyrnes

Nearly.  The algorithm turns up slightly different graphs each time (and
set.seed doesn't seem to make it consistent) and periodically chokes.  But
better than what I had.  Hrm.  I don't know much about the algorithm
graphviz uses for dot.  Do you have a reference on hand?  If it's simple,
I'd be willing to take a whack at it.


Gábor Csárdi-2 wrote:
 
 Jarrett,
 
 the 'igraph' package has a layout called layout.reingold.tilford that
 is designed for trees, there is a slight chance that it is good enough
 for you.
 
 Best,
 Gabor
 
 On Wed, Apr 29, 2009 at 10:11 PM, jebyrnes byr...@msi.ucsb.edu wrote:

 I've been using sna to work with some networks, and am trying to
 visualize
 them easily.  My networks are hierarchical (food webs).  All of the
 layout
 engines I've tried with gplot don't seem to plot hierarchical networks,
 as
 one would using dot from graphviz.  While I could do all of this by
 outputting to dotfiles and running it through graphviz, the graphics I
 get
 from R are much cleaner, and more easily integrated into my analyses.

 Is there any good way to diagram a hierarchical network in R, either with
 the sna library or otherwise?  It strikes me that at least the Netindices
 package can calculate trophic levels.  Could this be used for node
 placement?


 -Jarrett
 --
 View this message in context:
 http://www.nabble.com/Hierarchical-Diagram-of-Networks-in-sna-or-otherwise--tp23301819p23301819.html
 Sent from the R help mailing list archive at Nabble.com.

 __
 R-help@r-project.org mailing list
 https://stat.ethz.ch/mailman/listinfo/r-help
 PLEASE do read the posting guide
 http://www.R-project.org/posting-guide.html
 and provide commented, minimal, self-contained, reproducible code.

 
 
 
 -- 
 Gabor Csardi gabor.csa...@unil.ch UNIL DGM
 
 __
 R-help@r-project.org mailing list
 https://stat.ethz.ch/mailman/listinfo/r-help
 PLEASE do read the posting guide
 http://www.R-project.org/posting-guide.html
 and provide commented, minimal, self-contained, reproducible code.
 
 

-- 
View this message in context: 
http://www.nabble.com/Hierarchical-Diagram-of-Networks-in-sna-or-otherwise--tp23301819p23374024.html
Sent from the R help mailing list archive at Nabble.com.

__
R-help@r-project.org mailing list
https://stat.ethz.ch/mailman/listinfo/r-help
PLEASE do read the posting guide http://www.R-project.org/posting-guide.html
and provide commented, minimal, self-contained, reproducible code.


Re: [R] help_if command_new column

2009-05-04 Thread David Winsemius
You should be looking at the ifelse function. Ihe if and else  
constructs are not appropriate for recoding because they do not return  
vectors.


Much easier than all that folderol would be:

df$Aggregation - ifelse( df$Number1, School, Solitary)


On May 4, 2009, at 12:31 PM, barbara horta e costa wrote:


Hi everyone,

Do you know how to apply a If command to a vector in a data.frame,  
creating

a new column?

Example: If one species has 1 value in the Abundance column then  
it is

Solitary. If the abundance is higher than 1, is School.

I did:

aggregationFunction -function(x){
(if (x1) a- 'School'
   else a- 'Solitary')
   return (a)
  }

Then:

Aggregation - lapply (df$Number, aggregationFunction )



Then:

df - data.frame (df, Aggregation)



And I got this error:
Error in data.frame(df, Aggregation) :
 arguments imply differing number of rows: 2576, 1



Can you help me doing this function to a vector? I’ve tried to find  
this

question and the answer online but I couldn't.



I tried also only with if but it is only for 1 length and the error  
was:


 the condition has length  1 and only the first element will be used



This question seemed easy in the beginning but I was not successful.



Thanks a lot for helping me.

Barbara


--
Bárbara H. Costa
Marine Biologist Researcher
SCIAENA - Marine Sciences and Cooperation
www.sciaena.org

bco...@ispa.pt
barbarahco...@gmail.com

[[alternative HTML version deleted]]

__
R-help@r-project.org mailing list
https://stat.ethz.ch/mailman/listinfo/r-help
PLEASE do read the posting guide http://www.R-project.org/posting-guide.html
and provide commented, minimal, self-contained, reproducible code.


David Winsemius, MD
Heritage Laboratories
West Hartford, CT

__
R-help@r-project.org mailing list
https://stat.ethz.ch/mailman/listinfo/r-help
PLEASE do read the posting guide http://www.R-project.org/posting-guide.html
and provide commented, minimal, self-contained, reproducible code.


Re: [R] Surface for R outside of R

2009-05-04 Thread Francisco J. Zagmutt

Hi Koj,

I just completed a Windows application using the batch approach and it 
works very well.  In our case, we used VBA for Excel to call different 
batch files that execute R code, but you can do the same from any 
platform.  Here is a simple step-by-step example on how to make the 
batch files work (in Windows):



1. Write your R code and save it the directory where you want your 
results.  Here is a very simple example using a file called normal.r. 
This code takes 100 random samples from a standard normal and writes the 
results to a file called data.txt


write(rnorm(100),data.txt)
q(save=no, runLast = F)


2. Make sure that the console finds the path to R without changing your 
path environment variable.  The easiest way to do this is to download 
the batchfiles that Gabor Grothendieck has kindly provided in this site 
http://code.google.com/p/batchfiles/ and put one or all of the files in 
the same directory where your batch and R files will reside.  The only 
file I have needed so far in different WinXP and Vista machines was 
Rcmd.bat but you may need to use others.



3. Open a text editor and create a file with a .bat or .cmd extension. 
Here is an example of a file called TestBatch.CMD that runs the code 
in normal.r


@ECHO OFF
title Random number generation
ECHO Taking 100 random samples from a N(0,1)
Rcmd BATCH --slave normal.r Log.txt
ECHO Analysis done. See the file data.txt for results
PING 1.1.1.1 -n 1 -w 1000 NUL
@ECHO OFF

The main line here is Rcmd BATCH --slave normal.r Log.txt which tells 
the console to run your R code, and to write outputs printed in the 
console (i.e. errors) to the Log.txt file. The PING argument adds a 
short lag before the console closes, so the user can see what is echoed 
in the console. The other lines should be pretty self-explanatory.


4. Execute the TestBatch.CMD file (by hand or via your GUI) and watch 
the results.  You should now see two new files (data.txt and 
Log.txt) in the same directory as the batch file.



I hope this helps. Let me know if you have any other questions.

Regards,

Francisco

Francisco J. Zagmutt
Vose Consulting
2891 20th Street
Boulder, CO, 80304
USA
www.voseconsulting.com


Hans-Peter Suter wrote:

want an analysis. The best case were a surface of e.g. 8 Buttons, each click
leads to start a specific R file. My outputs are JPEG or CSV, so I don`t
need the output inside of R. Could anyone can give me some recommendations,
what could be a solution (e. g. Java)? Is such a solution possible?


 What about batch scripts which would be called by your GUI? (maybe
you could even skip the gui...)
(see ?Rscript (unix alike) or Windows FAQ, 2.12)

--
Regards,
Hans-Peter



__
R-help@r-project.org mailing list
https://stat.ethz.ch/mailman/listinfo/r-help
PLEASE do read the posting guide http://www.R-project.org/posting-guide.html
and provide commented, minimal, self-contained, reproducible code.


Re: [R] Nelson-Aalen estimator of cumulative hazard

2009-05-04 Thread Ravi Varadhan
Hi,

I figure out the reason for the difference.  There are ties in failure times
in the data set.  Consequently, it matters which method is used to handle
ties in coxph. The default is efron.  If I use method=breslow, there
is no difference between the 2 different ways of computing the Nelson-Aalen
estimte.

require(foreign)

gb - read.dta(GB.dta)  # Green  Byar data; N = 483

# Method 1 (note the use of Breslow estimator)

fit1 - coxph( Surv(time, status==Cancer | status==CVD |
status==Other) ~ 1, method=breslow, data=gb) 

h1 - basehaz(fit1)

# Method 2

fit2 - survfit(Surv(time, status==Cancer | status==CVD |
status==Other) ~ 1, data=gb)

jump - fit2$n.event  0

h2 -  cumsum(fit2$n.event[jump]/fit2$n.risk[jump])

 min(abs(h1$hazard - h2))
[1] 1.387779e-17



Best,
Ravi. 



---

Ravi Varadhan, Ph.D.

Assistant Professor, The Center on Aging and Health

Division of Geriatric Medicine and Gerontology 

Johns Hopkins University

Ph: (410) 502-2619

Fax: (410) 614-9625

Email: rvarad...@jhmi.edu

Webpage:  http://www.jhsph.edu/agingandhealth/People/Faculty/Varadhan.html

 





-Original Message-
From: r-help-boun...@r-project.org [mailto:r-help-boun...@r-project.org] On
Behalf Of Ravi Varadhan
Sent: Monday, May 04, 2009 12:33 AM
To: r-help@r-project.org
Subject: [R] Nelson-Aalen estimator of cumulative hazard

Hi,

I am computing the Nelson-Aalen (NA) estimate of baseline cumulative hazard
in two different ways using the survival package.  I am expecting that
they should be identical.  However, they are not. Their difference is a
monotonically increasing with time.  This difference is probably not large
to make any impact in the application, but is annoyingly non-trivial for me
to just ignore it.  

This is a competing risks problem, with the Green  Byar (1980) data set
(the STATA data set is attached).

Can anyone explain to me the reason for the discrepancy?


require(foreign)

gb - read.dta(GB.dta)  # Green  Byar data; N = 483

# Method 1

fit1 - coxph( Surv(time, status==Cancer | status==CVD |
status==Other) ~ 1, data=gb) 

h1 - basehaz(fit1)

# Method 2

fit2 - survfit(Surv(time, status==Cancer | status==CVD |
status==Other) ~ 1, data=gb)

jump - fit2$n.event  0

h2 -  cumsum(fit2$n.event[jump]/fit2$n.risk[jump])

plot(h1$time, h1$hazard - h2)

Thank you,
Ravi.


Ravi Varadhan, Ph.D.
Assistant Professor,
Division of Geriatric Medicine and Gerontology School of Medicine Johns
Hopkins University

Ph. (410) 502-2619
email: rvarad...@jhmi.edu

__
R-help@r-project.org mailing list
https://stat.ethz.ch/mailman/listinfo/r-help
PLEASE do read the posting guide http://www.R-project.org/posting-guide.html
and provide commented, minimal, self-contained, reproducible code.


[R] Producing a legend successfullly

2009-05-04 Thread Steve Murray

Dear all,

I'm attempting to insert a legend into a line graph. I've sorted out the 
positioning, but I'm unable to display the sample line and associated colour to 
go within the legend box. Instead, under the variable names, the numbers 1, 2, 
2, 3 are displayed in a column (with '2' repeated twice). This is the code I'm 
using:

legend(80,1150, c(Simulation, Observation, lty=1:2, col=2:3)

How do I go about displaying a red solid line next to 'Simulation' and a green 
dashed line next to 'Observation' (and if necessary, remove the numbers that 
are currently displayed)?

Many thanks for any help offered,

Steve

__
R-help@r-project.org mailing list
https://stat.ethz.ch/mailman/listinfo/r-help
PLEASE do read the posting guide http://www.R-project.org/posting-guide.html
and provide commented, minimal, self-contained, reproducible code.


[R] ellipsis problem

2009-05-04 Thread tyler
Hi,

I'm confused about the use of ellipsis in function arguments. I'm trying
to write a wrapper for plot to automate the combination of plot() and
points() calls for a data.frame. Some arguments seem to get passed
through to the inner plot, while others cause an error:

 Error in eval(expr, envir, enclos) : 
  ..1 used in an incorrect context, no ... to look in

As a minimal example:

tmp - data.frame(Y = sample(1:10, 40, replace = TRUE),
  X = sample(1:10, 40, replace = TRUE))

myplot - function(x, ...) {
  plot(Y ~ X, data = x, ...)
}

myplot(tmp) ## works fine
myplot(tmp, tcl = 1) ## works fine

myplot(tmp, tcl = -0.1)
Error in eval(expr, envir, enclos) : 
  ..1 used in an incorrect context, no ... to look in

myplot(tmp, mgp = c(3, 0.5, 0))
Error in eval(expr, envir, enclos) : 
  ..1 used in an incorrect context, no ... to look in

plot(Y ~ X, data = tmp, mgp = c(3, 0.5, 0)) ## works
plot(Y ~ X, data = tmp, tcl = -0.1) ## works

What am I doing wrong?

Thanks,

Tyler

R version 2.8.1 (2008-12-22)
Debian Testing

-- 
What is wanted is not the will to believe, but the will to find out,
which is the exact opposite.   --Bertrand Russell

__
R-help@r-project.org mailing list
https://stat.ethz.ch/mailman/listinfo/r-help
PLEASE do read the posting guide http://www.R-project.org/posting-guide.html
and provide commented, minimal, self-contained, reproducible code.


Re: [R] Code of the max() function

2009-05-04 Thread David Winsemius


On May 4, 2009, at 2:06 PM, Paul Smith wrote:


Dear All,

Where can one find the code of the max() function? Is that written  
in C?


pages 43-54 of:

http://cran.r-project.org/doc/Rnews/Rnews_2006-4.pdf

... is an article by Uwe Ligges on accesssing source code.

David Winsemius, MD
Heritage Laboratories
West Hartford, CT

__
R-help@r-project.org mailing list
https://stat.ethz.ch/mailman/listinfo/r-help
PLEASE do read the posting guide http://www.R-project.org/posting-guide.html
and provide commented, minimal, self-contained, reproducible code.


[R] [R-pkgs] frontier 0.99 is NOT backward compatible

2009-05-04 Thread Arne Henningsen
Dear current (and future) users of the frontier package,

We are approaching the first stable version (1.0) of the frontier package, 
which provides tools for microeconomic Stochastic Frontier Analysis (SFA). 
I have uploaded a kind of beta release (version 0.99) of this package to CRAN. 
The most important differences to version 0.9 affect the user interface. I 
have modified the package so that it is (hopefully) simpler to use and the 
functions are more similar to other R functions. Unfortunately, these changes 
make the new version NOT backward compatible. The most significant changes are 
reported below. A full ChangeLog is available in the log messages of the SVN 
repository on R-Forge [1]. Feedback about this package is highly appreciated. 
Please report any bugs, suggestions, feature requests, comments, and questions 
in the forums and trackers on R-Forge [1].

[1] http://r-forge.r-project.org/projects/frontier/

Best regards,
Arne


CHANGES IN VERSION 0.99-0

* corrected and considerably improved documentation of function fontier

* changed order of arguments of function frontier

* removed arguments crossSectionName and timePeriodName from function
frontier: in case of cross-section data, these variables (information) are
not needed, and in case of panel data, the information on the panel structure
has to be added to the data set by the function plm.data of the plm 
package

* argument modelType of function frontier must be ECF or EEF now

* replaced argument funtionType of function frontier
by argument ineffDecrease

* renamed argument eta of function frontier as timeEffect

* split argument mu of function frontier into truncNorm
and zIntercept

* increased default value of argument maxit of function frontier to 1000

* renamed argument step1 of function frontier as searchStep

* renamed argument tol2 of function frontier as searchTol

* replaced argument indic of function frontier by searchScale

* renamed argument gridno of function frontier as gridSize

* renamed argument igrid2 of function frontier as gridDouble

* renamed argument iprint of function frontier as printIter

* removed arguments qxNames and quadHalf from function frontier,
because quadratic or translog frontiers can be estimated with function
frontierQuad now

* added logLik method for objects of class frontier

* added function frontierQuad for estimating quadratic and translog 
frontiers

* added function frontierTranslogRay for estimating stochastic ray
translog frontier functions

* added the generic function efficiencies and corresponding methods
for default objects and objects of class frontier

* added elas method for objects of class frontierQuad

* function frontier() and frontierQuad() return the (matched) call now

* the matrix of efficiency estimates returned by frontier has row names
(cross section units) and possibly column names (time period) now


-- 
Arne Henningsen
http://www.arne-henningsen.name/

___
R-packages mailing list
r-packa...@r-project.org
https://stat.ethz.ch/mailman/listinfo/r-packages

__
R-help@r-project.org mailing list
https://stat.ethz.ch/mailman/listinfo/r-help
PLEASE do read the posting guide http://www.R-project.org/posting-guide.html
and provide commented, minimal, self-contained, reproducible code.


[R] rotate texts

2009-05-04 Thread phoebe kong
Hi all,

I'm hoping to add texts into a plot. I'm using text() to do it. I wonder if
the text() allow me to make the text vertically printed?

Thanks in advance for help,
phoebe

[[alternative HTML version deleted]]

__
R-help@r-project.org mailing list
https://stat.ethz.ch/mailman/listinfo/r-help
PLEASE do read the posting guide http://www.R-project.org/posting-guide.html
and provide commented, minimal, self-contained, reproducible code.


Re: [R] Hierarchical Diagram of Networks in sna or otherwise?

2009-05-04 Thread William Revelle

Jarrett,

At 11:31 AM -0700 5/4/09, jebyrnes wrote:

Nearly.  The algorithm turns up slightly different graphs each time (and
set.seed doesn't seem to make it consistent) and periodically chokes.  But
better than what I had.  Hrm.  I don't know much about the algorithm
graphviz uses for dot.  Do you have a reference on hand?  If it's simple,
I'd be willing to take a whack at it.


Gábor Csárdi-2 wrote:


 Jarrett,

 the 'igraph' package has a layout called layout.reingold.tilford that
 is designed for trees, there is a slight chance that it is good enough
 for you.

 Best,
 Gabor

 On Wed, Apr 29, 2009 at 10:11 PM, jebyrnes byr...@msi.ucsb.edu wrote:


 I've been using sna to work with some networks, and am trying to
 visualize
 them easily.  My networks are hierarchical (food webs).  All of the
 layout
 engines I've tried with gplot don't seem to plot hierarchical networks,
 as
 one would using dot from graphviz.  While I could do all of this by
 outputting to dotfiles and running it through graphviz, the graphics I
 get
 from R are much cleaner, and more easily integrated into my analyses.

 Is there any good way to diagram a hierarchical network in R, either with
 the sna library or otherwise?  It strikes me that at least the Netindices
 package can calculate trophic levels.  Could this be used for node
 placement?



  -Jarrett
 



If you like the dot output from graphviz you can 
get that using Rgraphviz from bioconductor. 
Although somewhat difficult to install, once 
installed it works beautifully.


Bill



--
William Revelle http://personality-project.org/revelle.html
Professor   http://personality-project.org/personality.html
Department of Psychology http://www.wcas.northwestern.edu/psych/
Northwestern University http://www.northwestern.edu/
Attend  ISSID/ARP:2009   http://issid.org/issid.2009/

__
R-help@r-project.org mailing list
https://stat.ethz.ch/mailman/listinfo/r-help
PLEASE do read the posting guide http://www.R-project.org/posting-guide.html
and provide commented, minimal, self-contained, reproducible code.


Re: [R] Surface for R outside of R

2009-05-04 Thread Gabor Grothendieck
Also see Rscript.bat in the same batchfiles distribution.  It
automatically finds Rscript.exe by looking in the registry
in the same way that Rcmd.bat finds Rcmd.exe.

On Mon, May 4, 2009 at 2:43 PM, Francisco J. Zagmutt
gerifalt...@hotmail.com wrote:
 Hi Koj,

 I just completed a Windows application using the batch approach and it works
 very well.  In our case, we used VBA for Excel to call different batch files
 that execute R code, but you can do the same from any platform.  Here is a
 simple step-by-step example on how to make the batch files work (in
 Windows):


 1. Write your R code and save it the directory where you want your results.
  Here is a very simple example using a file called normal.r. This code
 takes 100 random samples from a standard normal and writes the results to a
 file called data.txt

 write(rnorm(100),data.txt)
 q(save=no, runLast = F)


 2. Make sure that the console finds the path to R without changing your path
 environment variable.  The easiest way to do this is to download the
 batchfiles that Gabor Grothendieck has kindly provided in this site
 http://code.google.com/p/batchfiles/ and put one or all of the files in the
 same directory where your batch and R files will reside.  The only file I
 have needed so far in different WinXP and Vista machines was Rcmd.bat but
 you may need to use others.


 3. Open a text editor and create a file with a .bat or .cmd extension. Here
 is an example of a file called TestBatch.CMD that runs the code in
 normal.r

 @ECHO OFF
 title Random number generation
 ECHO Taking 100 random samples from a N(0,1)
 Rcmd BATCH --slave normal.r Log.txt
 ECHO Analysis done. See the file data.txt for results
 PING 1.1.1.1 -n 1 -w 1000 NUL
 @ECHO OFF

 The main line here is Rcmd BATCH --slave normal.r Log.txt which tells the
 console to run your R code, and to write outputs printed in the console
 (i.e. errors) to the Log.txt file. The PING argument adds a short lag before
 the console closes, so the user can see what is echoed in the console. The
 other lines should be pretty self-explanatory.

 4. Execute the TestBatch.CMD file (by hand or via your GUI) and watch the
 results.  You should now see two new files (data.txt and Log.txt) in the
 same directory as the batch file.


 I hope this helps. Let me know if you have any other questions.

 Regards,

 Francisco

 Francisco J. Zagmutt
 Vose Consulting
 2891 20th Street
 Boulder, CO, 80304
 USA
 www.voseconsulting.com


 Hans-Peter Suter wrote:

 want an analysis. The best case were a surface of e.g. 8 Buttons, each
 click
 leads to start a specific R file. My outputs are JPEG or CSV, so I don`t
 need the output inside of R. Could anyone can give me some
 recommendations,
 what could be a solution (e. g. Java)? Is such a solution possible?

  What about batch scripts which would be called by your GUI? (maybe
 you could even skip the gui...)
 (see ?Rscript (unix alike) or Windows FAQ, 2.12)

 --
 Regards,
 Hans-Peter


 __
 R-help@r-project.org mailing list
 https://stat.ethz.ch/mailman/listinfo/r-help
 PLEASE do read the posting guide http://www.R-project.org/posting-guide.html
 and provide commented, minimal, self-contained, reproducible code.


__
R-help@r-project.org mailing list
https://stat.ethz.ch/mailman/listinfo/r-help
PLEASE do read the posting guide http://www.R-project.org/posting-guide.html
and provide commented, minimal, self-contained, reproducible code.


Re: [R] Producing a legend successfullly

2009-05-04 Thread Peter Flom
Steve Murray smurray...@hotmail.com wrote

I'm attempting to insert a legend into a line graph. I've sorted out the 
positioning, but I'm unable to display the sample line and associated colour 
to go within the legend box. Instead, under the variable names, the numbers 1, 
2, 2, 3 are displayed in a column (with '2' repeated twice). This is the code 
I'm using:

legend(80,1150, c(Simulation, Observation, lty=1:2, col=2:3)


You've got mismatched parentheses here, with two ( and only one ).

I suspect you added one at the end, making r look at lty= and col = as part of 
the legend text.

You want

legend(80,1150, c(Simulation, Observation), lty=1:2, col=2:3)

HTH

Peter

Peter L. Flom, PhD
Statistical Consultant
www DOT peterflomconsulting DOT com

__
R-help@r-project.org mailing list
https://stat.ethz.ch/mailman/listinfo/r-help
PLEASE do read the posting guide http://www.R-project.org/posting-guide.html
and provide commented, minimal, self-contained, reproducible code.


Re: [R] Producing a legend successfullly

2009-05-04 Thread Sarah Goslee
This works for me:
 plot(1:10, 1:10)
 legend(topleft, c(Simulation, Observation), lty=1:2, col=2:3)

Try that; if it does work, then we need more information about
exactly what you're doing, because there's a problem not contained
in just that single line of code (which has a problem of its own!).

Sarah

On Mon, May 4, 2009 at 2:46 PM, Steve Murray smurray...@hotmail.com wrote:

 Dear all,

 I'm attempting to insert a legend into a line graph. I've sorted out the 
 positioning, but I'm unable to display the sample line and associated colour 
 to go within the legend box. Instead, under the variable names, the numbers 
 1, 2, 2, 3 are displayed in a column (with '2' repeated twice). This is the 
 code I'm using:

 legend(80,1150, c(Simulation, Observation, lty=1:2, col=2:3)

 How do I go about displaying a red solid line next to 'Simulation' and a 
 green dashed line next to 'Observation' (and if necessary, remove the numbers 
 that are currently displayed)?


-- 
Sarah Goslee
http://www.functionaldiversity.org

__
R-help@r-project.org mailing list
https://stat.ethz.ch/mailman/listinfo/r-help
PLEASE do read the posting guide http://www.R-project.org/posting-guide.html
and provide commented, minimal, self-contained, reproducible code.


Re: [R] Producing a legend successfullly

2009-05-04 Thread Jorge Ivan Velez
Dear Steve,
Try this:
x-rnorm(100)
y-rnorm(100) + 2*x
plot(x,y)
legend('topleft', c(Simulation, Observation), lty=1:2, col=2:3)
abline(x,y,col=3,lty=2)

(you forgot a ) in your code :( )

HTH,

Jorge


On Mon, May 4, 2009 at 2:46 PM, Steve Murray smurray...@hotmail.com wrote:


 Dear all,

 I'm attempting to insert a legend into a line graph. I've sorted out the
 positioning, but I'm unable to display the sample line and associated colour
 to go within the legend box. Instead, under the variable names, the numbers
 1, 2, 2, 3 are displayed in a column (with '2' repeated twice). This is the
 code I'm using:

 legend(80,1150, c(Simulation, Observation, lty=1:2, col=2:3)

 How do I go about displaying a red solid line next to 'Simulation' and a
 green dashed line next to 'Observation' (and if necessary, remove the
 numbers that are currently displayed)?

 Many thanks for any help offered,

 Steve

 __
 R-help@r-project.org mailing list
 https://stat.ethz.ch/mailman/listinfo/r-help
 PLEASE do read the posting guide
 http://www.R-project.org/posting-guide.html
 and provide commented, minimal, self-contained, reproducible code.


[[alternative HTML version deleted]]

__
R-help@r-project.org mailing list
https://stat.ethz.ch/mailman/listinfo/r-help
PLEASE do read the posting guide http://www.R-project.org/posting-guide.html
and provide commented, minimal, self-contained, reproducible code.


Re: [R] ellipsis problem

2009-05-04 Thread Duncan Murdoch

On 5/4/2009 2:55 PM, tyler wrote:

Hi,

I'm confused about the use of ellipsis in function arguments. I'm trying
to write a wrapper for plot to automate the combination of plot() and
points() calls for a data.frame. Some arguments seem to get passed
through to the inner plot, while others cause an error:


This looks like another manifestation of the following bug:

o   The ... argument was not handled properly when ... was found
in the enclosure of the current function, rather than in the
function header itself.  (This caused integrate() to fail in
certain cases.)

which has recently been fixed in R-patched.  I haven't traced through 
it, but I do see the same error as you in 2.9.0, but not in 2.9.0 patched.


Duncan Murdoch



 Error in eval(expr, envir, enclos) : 
  ..1 used in an incorrect context, no ... to look in


As a minimal example:

tmp - data.frame(Y = sample(1:10, 40, replace = TRUE),
  X = sample(1:10, 40, replace = TRUE))

myplot - function(x, ...) {
  plot(Y ~ X, data = x, ...)
}

myplot(tmp) ## works fine
myplot(tmp, tcl = 1) ## works fine

myplot(tmp, tcl = -0.1)
Error in eval(expr, envir, enclos) : 
  ..1 used in an incorrect context, no ... to look in


myplot(tmp, mgp = c(3, 0.5, 0))
Error in eval(expr, envir, enclos) : 
  ..1 used in an incorrect context, no ... to look in


plot(Y ~ X, data = tmp, mgp = c(3, 0.5, 0)) ## works
plot(Y ~ X, data = tmp, tcl = -0.1) ## works

What am I doing wrong?


I think the only thing you are doing wrong is not keeping up to date 
with the latest patches.


Duncan Murdoch



Thanks,

Tyler

R version 2.8.1 (2008-12-22)
Debian Testing



__
R-help@r-project.org mailing list
https://stat.ethz.ch/mailman/listinfo/r-help
PLEASE do read the posting guide http://www.R-project.org/posting-guide.html
and provide commented, minimal, self-contained, reproducible code.


Re: [R] Code of the max() function

2009-05-04 Thread Paul Smith
On Mon, May 4, 2009 at 7:47 PM, David Winsemius dwinsem...@comcast.net wrote:
 Where can one find the code of the max() function? Is that written in C?

 pages 43-54 of:

 http://cran.r-project.org/doc/Rnews/Rnews_2006-4.pdf

 ... is an article by Uwe Ligges on accesssing source code.

Thanks, David. That is it!

Paul

__
R-help@r-project.org mailing list
https://stat.ethz.ch/mailman/listinfo/r-help
PLEASE do read the posting guide http://www.R-project.org/posting-guide.html
and provide commented, minimal, self-contained, reproducible code.


Re: [R] Producing a legend successfullly

2009-05-04 Thread baptiste auguie

your code is missing a closing bracket for the text labels,

 legend(1,4, c(Simulation, Observation), lty=1:2, col=2:3)

baptiste

On 4 May 2009, at 20:46, Steve Murray wrote:



Dear all,

I'm attempting to insert a legend into a line graph. I've sorted out  
the positioning, but I'm unable to display the sample line and  
associated colour to go within the legend box. Instead, under the  
variable names, the numbers 1, 2, 2, 3 are displayed in a column  
(with '2' repeated twice). This is the code I'm using:


legend(80,1150, c(Simulation, Observation, lty=1:2, col=2:3)

How do I go about displaying a red solid line next to 'Simulation'  
and a green dashed line next to 'Observation' (and if necessary,  
remove the numbers that are currently displayed)?


Many thanks for any help offered,

Steve

__
R-help@r-project.org mailing list
https://stat.ethz.ch/mailman/listinfo/r-help
PLEASE do read the posting guide http://www.R-project.org/posting-guide.html
and provide commented, minimal, self-contained, reproducible code.


_

Baptiste Auguié

School of Physics
University of Exeter
Stocker Road,
Exeter, Devon,
EX4 4QL, UK

Phone: +44 1392 264187

http://newton.ex.ac.uk/research/emag

__
R-help@r-project.org mailing list
https://stat.ethz.ch/mailman/listinfo/r-help
PLEASE do read the posting guide http://www.R-project.org/posting-guide.html
and provide commented, minimal, self-contained, reproducible code.


Re: [R] rotate texts

2009-05-04 Thread David Winsemius


On May 4, 2009, at 3:04 PM, phoebe kong wrote:


Hi all,

I'm hoping to add texts into a plot. I'm using text() to do it. I  
wonder if

the text() allow me to make the text vertically printed?


This worked in a plot I hapened to have lying around:

text(1,1,test, srt=90)




David Winsemius, MD
Heritage Laboratories
West Hartford, CT

__
R-help@r-project.org mailing list
https://stat.ethz.ch/mailman/listinfo/r-help
PLEASE do read the posting guide http://www.R-project.org/posting-guide.html
and provide commented, minimal, self-contained, reproducible code.


[R] Calculating relative ratios in a data frame..

2009-05-04 Thread Zeljko Vrba
I have a data-set that is structured as follows:

 spwgn v.realtime v.cputime v.tcputime  v.idletime v.nswtch
9 0 11   28.61300 28.6128.6039 0.0e+00  407
1563  0 12   15.20270 30.3828.5981 9.80523e-01  483
3128  0 14   12.50930 50.0028.6053 1.07877e+01  489
4682  0 16   12.10260 72.5528.6193 2.20203e+01  488
6241  0 18   12.11510 96.8028.6448 3.41126e+01  494
121   0101   28.61530 28.6028.6056 0.0e+00 1996
1684  0102   14.33530 28.6628.6003 2.96437e-02 1908
3241  01047.27129 29.0528.6002 2.31302e-01 2110
4801  01064.91522 29.4228.6002 4.25931e-01 2134
6367  01083.79337 30.2528.6032 8.42412e-01 2245

[And a lot of other variables, and sp also changes]

Now, I want to divide each of the v.* variables for every (sp, wg, n)
combination by the values in the row defined by (sp, wg, 1), thus giving 
a relative change to the case n==1 for the same combination of sp and wg.
I have written the following function, but I'm wondering whether there's
an easier/more efficient way..  Perhaps by organizing the data structure
differently, and if so -- how?

speedup.1 - function(df)
{
  groups - split(df, df[,c(sp,wg)])
  ret - list()

  for(g in groups) {
ref - g[1,]
for(v in 4:15)  # sp,wg,n are factors, so / doesn't work..
  g[[v]] - ref[[v]] / g[[v]]
ret - rbind(ret, g)
  }
  ret
}

__
R-help@r-project.org mailing list
https://stat.ethz.ch/mailman/listinfo/r-help
PLEASE do read the posting guide http://www.R-project.org/posting-guide.html
and provide commented, minimal, self-contained, reproducible code.


[R] sm.density.compare

2009-05-04 Thread Claudia Penaloza
I've been using the sm.denstiy.compare function and would like to know some
specifics:
a) What kind of bootstrap sampling is used, with or without replacement?

b) A p-value is returned after running the following script (for example):

library(sm)
y - rnorm(100)
g - rep(1:2, rep(50,2))
sm.density.compare(y, g, model=equal)

to what statistic does this p-value correspond?

c) Also, in the help file this description appears for Value of
sm.density.compareomparison of univariate density estimates

When model is set to none, nothing is returned. When model is set to
equal, a list containing the smoothing parameter and the p-value of the
test is returned. When band takes the value TRUE, and there are only two
groups to compare, the list contains in addition the upper and lower
end-points of the reference band for equality.

however, the mentioned list is not returned when I run the analysis...
only the p-value is returned.
Am I doing something wrong?

Any help is appreciated.
Thank you,
Claudia

[[alternative HTML version deleted]]

__
R-help@r-project.org mailing list
https://stat.ethz.ch/mailman/listinfo/r-help
PLEASE do read the posting guide http://www.R-project.org/posting-guide.html
and provide commented, minimal, self-contained, reproducible code.


[R] to the person who asked about dividing by the base row

2009-05-04 Thread markleeds

   to whomever that was. i deleted your email but I think below does what you
   want. as always, there's probably some
   improvement that could be done. whether it's  minor or major, i'm not sure
   ?
   also, 3 things:Â Â
   A)there are some Inf's in the output because of division by zero but I
   wasn't sure how you wanted to handle that.
   B) it also assumes that an n=1 always exists and you should probably have
   some error checking in there for when it doesn't.Â
   C) When i tried to condense it to below, I got an error that division is
   only valid for equal sized data frames and
   I'm not sure why so maybe someone can say something about that ?
   temp - lapply(split(DF,list(DF$sp,DF$wg)), function(.df) {
   Â Â Â .df[(.df$n!=1),c(3,4,5,6,7)]-   .df[(.df$n   !=
   1),c(3,4,5,6,7)]/.df[(.df$n == 1),c(3,4,5,6,7)]
   Â Â Â  .df
   })
   # WORKING CODE
   #===
   Â
   DF - read.table(textConnection(sp wg n v.realtime v.cputime v.tcputime
   v.idletime v.nswtch
   9 0 1 1 28.61300 28.61 28.6039 0.0e+00 407
   1563 0 1 2 15.20270 30.38 28.5981 9.80523e-01 483
   3128 0 1 4 12.50930 50.00 28.6053 1.07877e+01 489
   4682 0 1 6 12.10260 72.55 28.6193 2.20203e+01 488
   6241 0 1 8 12.11510 96.80 28.6448 3.41126e+01 494
   121 0 10 1 28.61530 28.60 28.6056 0.0e+00 1996
   1684 0 10 2 14.33530 28.66 28.6003 2.96437e-02 1908
   3241 0 10 4 7.27129 29.05 28.6002 2.31302e-01 2110
   4801 0 10 6 4.91522 29.42 28.6002 4.25931e-01 2134
   6367 0 10 8 3.79337 30.25 28.6032 8.42412e-01 2245),header=TRUE)

   print(DF)

   temp - lapply(split(DF,list(DF$sp,DF$wg)), function(.df) {
   Â Â Â.df$v.realtime[.df$n   !=   1]   -  .df$v.realtime[.df$n  !=
   1]/.df$v.realtime[.df$n == 1]
   Â Â Â.df$v.cputime[.df$n   !=   1]   -   .df$v.cputime[.df$n   !=
   1]/.df$v.cputime[.df$n == 1]
     Â.df$v.tcputime[.df$n   !=  1]  -  .df$v.tcputime[.df$n   !=
   1]/.df$v.tcputime[.df$n == 1]
   Â Â Â.df$v.idletime[.df$n   !=   1]   -  .df$v.idletime[.df$n  !=
   1]/.df$v.idletime[.df$n == 1]
   Â Â Â .df$v.nswtch[.df$n!=   1]   -   .df$v.nswtch[.df$n   !=
   1]/.df$v.nswtch[.df$n == 1]
   Â Â Â  .df
   })

   result - do.call(rbind,temp)
   rownames(result) - NULL
   print(result)
__
R-help@r-project.org mailing list
https://stat.ethz.ch/mailman/listinfo/r-help
PLEASE do read the posting guide http://www.R-project.org/posting-guide.html
and provide commented, minimal, self-contained, reproducible code.


Re: [R] Calculating relative ratios in a data frame..

2009-05-04 Thread jim holtman
Here is a way -- you are dividing by zero in the scaling:

 x
 sp wg n v.realtime v.cputime v.tcputime v.idletime v.nswtch
9 0  1 1   28.61300 28.6128.6039  0.000  407
1563  0  1 2   15.20270 30.3828.5981  0.9805230  483
3128  0  1 4   12.50930 50.0028.6053 10.7877000  489
4682  0  1 6   12.10260 72.5528.6193 22.0203000  488
6241  0  1 8   12.11510 96.8028.6448 34.1126000  494
121   0 10 1   28.61530 28.6028.6056  0.000 1996
1684  0 10 2   14.33530 28.6628.6003  0.0296437 1908
3241  0 10 47.27129 29.0528.6002  0.2313020 2110
4801  0 10 64.91522 29.4228.6002  0.4259310 2134
6367  0 10 83.79337 30.2528.6032  0.8424120 2245
 x.s - split(x, x[,1:2])
 x.result - do.call(rbind, lapply(x.s, function(.sec){
+ # create matrix of data to scale
+ .mat - as.matrix(.sec[, -(1:3)])
+ # need to transpose due to recycle of arguments
+ .mat - t(t(.mat) / .mat[1,])
+ .sec[, -(1:3)] - as.data.frame(.mat)
+ .sec
+ }))
 x.result
  sp wg n v.realtime v.cputime v.tcputime v.idletime  v.nswtch
0.1.9  0  1 1  1.000  1.00  1.000NaN 1.000
0.1.1563   0  1 2  0.5313214  1.061866  0.9997972Inf 1.1867322
0.1.3128   0  1 4  0.4371894  1.747641  1.489Inf 1.2014742
0.1.4682   0  1 6  0.4229756  2.535827  1.0005384Inf 1.1990172
0.1.6241   0  1 8  0.4234124  3.383432  1.0014299Inf 1.2137592
0.10.121   0 10 1  1.000  1.00  1.000NaN 1.000
0.10.1684  0 10 2  0.5009663  1.002098  0.9998147Inf 0.9559118
0.10.3241  0 10 4  0.2541050  1.015734  0.9998112Inf 1.0571142
0.10.4801  0 10 6  0.1717689  1.028671  0.9998112Inf 1.0691383
0.10.6367  0 10 8  0.1325644  1.057692  0.161Inf 1.1247495




On Mon, May 4, 2009 at 3:41 PM, Zeljko Vrba zv...@ifi.uio.no wrote:

 I have a data-set that is structured as follows:

 spwgn v.realtime v.cputime v.tcputime  v.idletime v.nswtch
 9 0 11   28.61300 28.6128.6039 0.0e+00  407
 1563  0 12   15.20270 30.3828.5981 9.80523e-01  483
 3128  0 14   12.50930 50.0028.6053 1.07877e+01  489
 4682  0 16   12.10260 72.5528.6193 2.20203e+01  488
 6241  0 18   12.11510 96.8028.6448 3.41126e+01  494
 121   0101   28.61530 28.6028.6056 0.0e+00 1996
 1684  0102   14.33530 28.6628.6003 2.96437e-02 1908
 3241  01047.27129 29.0528.6002 2.31302e-01 2110
 4801  01064.91522 29.4228.6002 4.25931e-01 2134
 6367  01083.79337 30.2528.6032 8.42412e-01 2245

 [And a lot of other variables, and sp also changes]

 Now, I want to divide each of the v.* variables for every (sp, wg, n)
 combination by the values in the row defined by (sp, wg, 1), thus giving
 a relative change to the case n==1 for the same combination of sp and wg.
 I have written the following function, but I'm wondering whether there's
 an easier/more efficient way..  Perhaps by organizing the data structure
 differently, and if so -- how?

 speedup.1 - function(df)
 {
  groups - split(df, df[,c(sp,wg)])
  ret - list()

  for(g in groups) {
ref - g[1,]
for(v in 4:15)  # sp,wg,n are factors, so / doesn't work..
  g[[v]] - ref[[v]] / g[[v]]
ret - rbind(ret, g)
  }
  ret
 }

 __
 R-help@r-project.org mailing list
 https://stat.ethz.ch/mailman/listinfo/r-help
 PLEASE do read the posting guide
 http://www.R-project.org/posting-guide.htmlhttp://www.r-project.org/posting-guide.html
 and provide commented, minimal, self-contained, reproducible code.




-- 
Jim Holtman
Cincinnati, OH
+1 513 646 9390

What is the problem that you are trying to solve?

[[alternative HTML version deleted]]

__
R-help@r-project.org mailing list
https://stat.ethz.ch/mailman/listinfo/r-help
PLEASE do read the posting guide http://www.R-project.org/posting-guide.html
and provide commented, minimal, self-contained, reproducible code.


Re: [R] can install.packages() copy utility files to the public_html directory ?

2009-05-04 Thread Uwe Ligges



Markus Loecher wrote:

Dear fellow R-users,
I am about to publish an HTML utility package to CRAN that expands on the
R2HTML package and includes a few goodies such as sorted tables, easy
automation of framed HTML reporting, etc.
However, some of the resulting dynamic HTML pages need to access JavaScript
code that should sit in a specific subdirectory of public_html.
My more general question is hence, (i) how do I include the directory
containing the JavaScript code in my R package and 


Yes, in directory ./inst/JavaScript for example that is copied to 
./JavaScript during installation of the package.




(ii) is it possible to
copy this directory to the user's public_html path during installation ?


Yes, if it is a source package (not for a binary package), BUT you 
should not write such code that copyies files to a directory different 
than the package's directory in the library that it is installed to. 
Things that install files to suprising locations are sometimes 
considered to be some kind of malware.


Uwe Ligges





Thanks!
Markus

[[alternative HTML version deleted]]

__
R-help@r-project.org mailing list
https://stat.ethz.ch/mailman/listinfo/r-help
PLEASE do read the posting guide http://www.R-project.org/posting-guide.html
and provide commented, minimal, self-contained, reproducible code.


__
R-help@r-project.org mailing list
https://stat.ethz.ch/mailman/listinfo/r-help
PLEASE do read the posting guide http://www.R-project.org/posting-guide.html
and provide commented, minimal, self-contained, reproducible code.


[R] ellipse

2009-05-04 Thread Antonio Lucadamo

Dear all,
I'm using the ellipse package and I would like to verify if the  
confidence region that I
build with this package can be equivalent to an Union Intersection  
Test. I used different

value for the t-statistic but I can not find the right equivalence.
Does someone know how to choose the right value?

Thanks a lot.
Antonio


--
Antonio Lucadamo,
Dipartimento di Scienze Economiche e Metodi Quantitativi
Università del Piemonte Orientale A. Avogadro
via Perrone, 18 - 28100 Novara
fax:  +39 0321 375305
phone:+39 0321 375338
mobile phone: +39 3385973881
skype contact: antonio.lucadamo

__
R-help@r-project.org mailing list
https://stat.ethz.ch/mailman/listinfo/r-help
PLEASE do read the posting guide http://www.R-project.org/posting-guide.html
and provide commented, minimal, self-contained, reproducible code.


[R] A variation on the bar plot

2009-05-04 Thread dxc13

Hi all,
I cannot think of the technical name of this plot, but I want to create a
plot with the data below that looks like two back-to-back horizontal bar
plots.
Ideally, there would be a vertical line in the center of the plot at zero,
and on the right hand side would be 4 bars representing the values for 2007,
and on the left side of the vertical line would be the corresponding values
for 2005.  Even though values would be on the left side of the zero line, it
doesn't mean they are negative, I just want to compare changes in time.

data
 2005  2007
11.42 11.76
11.93 11.97
4.06   4.53 
4.29   4.47 

Hope this is clear,
Thanks in advance.

dxc13
-- 
View this message in context: 
http://www.nabble.com/A-variation-on-the-bar-plot-tp23374466p23374466.html
Sent from the R help mailing list archive at Nabble.com.

__
R-help@r-project.org mailing list
https://stat.ethz.ch/mailman/listinfo/r-help
PLEASE do read the posting guide http://www.R-project.org/posting-guide.html
and provide commented, minimal, self-contained, reproducible code.


Re: [R] Optim function in the loop

2009-05-04 Thread Rstarer

Thanks for the reply. 

optim(initpar,Linn,NULL,method=BFGS,hessian=TRUE,control=list(trace=1,REPORT=1,maxit=300))

The optim is to minimize the negative likelihood function which is performed
in function Linn,
Every time the code stops right after optim, for example, the output looks
like:


iter  16 value 42.818394
iter  16 value 42.818394
final  value 42.818394 
converged
 

It seems from the code that the loop should continue to run, at least
calculate u values... but it does not...

Thank you.


Hi all, 
I wrote the following lines of codes try to do some iterations to find the
global optimal values, but the function does not execute properly. Every
time codes stop after one iteration right after executing the optim()
function. Does anyone could have me to take a look? Thanks.
if (count0){

k=k+0.05;

mu0=c(83+k,0,0)

Sigma0= diag(0.4,3)

initpar=c(.1+10*k,10*k,10*k,10*k) # initial parameters for Phi[1,1], the 2
Q’s and R

est=optim(initpar,Linn,NULL,method=BFGS,hessian=TRUE,control=list(trace=1,REPORT=1,maxit=300))

stderr=sqrt(diag(solve(est$hessian)))

estimate=est$par

u=cbind(estimate,stderr)

if (u[1]0  u[2]0  u[3]0  u[4]0){

print(mu0=);print(mu0);

print(initpar=);print(initpar)

#break

} else {count=-1;

}

}



-- 
View this message in context: 
http://www.nabble.com/Optim-function-in-the-loop-tp23352044p23374525.html
Sent from the R help mailing list archive at Nabble.com.

__
R-help@r-project.org mailing list
https://stat.ethz.ch/mailman/listinfo/r-help
PLEASE do read the posting guide http://www.R-project.org/posting-guide.html
and provide commented, minimal, self-contained, reproducible code.


[R] FOURIER INTEGRALS IN R

2009-05-04 Thread Achilleas Achilleos
Hi,

I am wondering whether there exist any function in R (any package) that
calculates Fourier Integrals.

Particularly, I am interested for estimation of a Cosine Fourier integral...

I would be much obliged if you could help me on this..

Thanks.

Andreas




--

__
R-help@r-project.org mailing list
https://stat.ethz.ch/mailman/listinfo/r-help
PLEASE do read the posting guide http://www.R-project.org/posting-guide.html
and provide commented, minimal, self-contained, reproducible code.


Re: [R] A variation on the bar plot

2009-05-04 Thread Gabor Grothendieck
Check out the back to back histogram here:
http://addictedtor.free.fr/graphiques/RGraphGallery.php?graph=136


On Mon, May 4, 2009 at 2:58 PM, dxc13 dx...@health.state.ny.us wrote:

 Hi all,
 I cannot think of the technical name of this plot, but I want to create a
 plot with the data below that looks like two back-to-back horizontal bar
 plots.
 Ideally, there would be a vertical line in the center of the plot at zero,
 and on the right hand side would be 4 bars representing the values for 2007,
 and on the left side of the vertical line would be the corresponding values
 for 2005.  Even though values would be on the left side of the zero line, it
 doesn't mean they are negative, I just want to compare changes in time.

data
  2005  2007
 11.42 11.76
 11.93 11.97
 4.06   4.53
 4.29   4.47

 Hope this is clear,
 Thanks in advance.

 dxc13
 --
 View this message in context: 
 http://www.nabble.com/A-variation-on-the-bar-plot-tp23374466p23374466.html
 Sent from the R help mailing list archive at Nabble.com.

 __
 R-help@r-project.org mailing list
 https://stat.ethz.ch/mailman/listinfo/r-help
 PLEASE do read the posting guide http://www.R-project.org/posting-guide.html
 and provide commented, minimal, self-contained, reproducible code.


__
R-help@r-project.org mailing list
https://stat.ethz.ch/mailman/listinfo/r-help
PLEASE do read the posting guide http://www.R-project.org/posting-guide.html
and provide commented, minimal, self-contained, reproducible code.


[R] questions about function arima0

2009-05-04 Thread Beáta Darabos
Hi,

I work on order estimation for autoregressive processes and after some
inconsistencies cropped up I implemented the AIC criterion myself. Its
results do not match the implementation in R and there are a few
things I can not understand even after reading the source code of R.

I used the function called arima0 (with empty ma coeficient vector),
and I do not understand how some of the results are calculated:

1. resid

If we denote the estimated order of a time series by k, how does the
function calculate the first k elements of the residual vector? The
rest of the elements match my version, but the first k are not
calculated in my implementation. Calculating them requires some kind
of assumption, but after trying a few possible ones (like using zero
in place of the missing elements ot the average of the time series)
none of them seem to yield the same results as I get from arima0.

2. loglik

The loglikelihood at it's maximum should be a funcion of the estimated
residual variance if the residuals are assumed to be normally
distributed:

-2*loglik = n*log(sigma2)+n*log(2*pi)+n

The $loglik and $sigma2 variables from arima0 do not satisfy that equation.

In my implementation I calculated the residual vector the same way,
but leaving the first k elements, then calculating sigma2 by
sum(resid^2)/(n-k), and at the end I got the loglikelihood through the
equation above.
If I compare the Akaike information criteria I get from my
implementation to the one of R's, I get quite different results.
I performed 100 simulations with time series of length 1000 and the
ratio of correct order selection was different (lower with my
implementation than with R's).

Can you help me to find out what causes the difference?

Best regards,
Beata Darabos

__
R-help@r-project.org mailing list
https://stat.ethz.ch/mailman/listinfo/r-help
PLEASE do read the posting guide http://www.R-project.org/posting-guide.html
and provide commented, minimal, self-contained, reproducible code.


Re: [R] A variation on the bar plot

2009-05-04 Thread S Ellison
It's sometimes called a pyramid plot.

The plotrix package has one. See ?pyramid.plot



 dxc13 dx...@health.state.ny.us 05/04/09 7:58 PM 

Hi all,
I cannot think of the technical name of this plot, but I want to create
a
plot with the data below that looks like two back-to-back horizontal bar
plots.
Ideally, there would be a vertical line in the center of the plot at
zero,
and on the right hand side would be 4 bars representing the values for
2007,
and on the left side of the vertical line would be the corresponding
values
for 2005.  Even though values would be on the left side of the zero
line, it
doesn't mean they are negative, I just want to compare changes in time.

data
 2005  2007
11.42 11.76
11.93 11.97
4.06   4.53 
4.29   4.47 

Hope this is clear,
Thanks in advance.

dxc13
-- 
View this message in context:
http://www.nabble.com/A-variation-on-the-bar-plot-tp23374466p23374466.html
Sent from the R help mailing list archive at Nabble.com.

__
R-help@r-project.org mailing list
https://stat.ethz.ch/mailman/listinfo/r-help
PLEASE do read the posting guide
http://www.R-project.org/posting-guide.html
and provide commented, minimal, self-contained, reproducible code.


***
This email and any attachments are confidential. Any use...{{dropped:8}}

__
R-help@r-project.org mailing list
https://stat.ethz.ch/mailman/listinfo/r-help
PLEASE do read the posting guide http://www.R-project.org/posting-guide.html
and provide commented, minimal, self-contained, reproducible code.


Re: [R] ellipsis problem

2009-05-04 Thread tyler
Duncan Murdoch murd...@stats.uwo.ca writes:

 On 5/4/2009 2:55 PM, tyler wrote:


 This looks like another manifestation of the following bug:

[...]

 which has recently been fixed in R-patched. I haven't traced through
 it, but I do see the same error as you in 2.9.0, but not in 2.9.0
 patched.


Ok, thanks. I've updated to 2.9.0, but we don't have the patched version
packaged for Debian yet. I can work around the problem until we do.

Cheers,

Tyler

-- 
Philosophy of science is about as useful to scientists as ornithology is
to birds.  --Richard Feynman

__
R-help@r-project.org mailing list
https://stat.ethz.ch/mailman/listinfo/r-help
PLEASE do read the posting guide http://www.R-project.org/posting-guide.html
and provide commented, minimal, self-contained, reproducible code.


[R] S4 slot of type connection [Sec=Unclassified]

2009-05-04 Thread Troy Robertson
I am having trouble declaring a slot of type connection or file in which to 
store a file connection.

I get a warning message when class definition is loaded:



In .completeClassSlots(ClassDef, where) :

  undefined slot classes in definition of Element: datafileConn(class file)



Can anyone suggest what type I should be using for the slot.  Type connection 
produces similar warnings plus error messages.





Troy Robertson

Database and Computing Support Provider

Southern Ocean Ecosystems, ERM/Fish

Australian Antarctic Division

Channel Highway, Kingston 7050

PH: 03 62323571

troy.robert...@aad.gov.au




___

Australian Antarctic Division - Commonwealth of Australia
IMPORTANT: This transmission is intended for the addressee only. If you are not 
the
intended recipient, you are notified that use or dissemination of this 
communication is
strictly prohibited by Commonwealth law. If you have received this transmission 
in error,
please notify the sender immediately by e-mail or by telephoning +61 3 6232 
3209 and
DELETE the message.
Visit our web site at http://www.antarctica.gov.au/
___

[[alternative HTML version deleted]]

__
R-help@r-project.org mailing list
https://stat.ethz.ch/mailman/listinfo/r-help
PLEASE do read the posting guide http://www.R-project.org/posting-guide.html
and provide commented, minimal, self-contained, reproducible code.


Re: [R] FOURIER INTEGRALS IN R

2009-05-04 Thread andrew
integrate offers some one-dimensional algorithms, but you need to
start with a smooth function to get it to converge properly.  With a
cosine integral, there may be certain routines that offer better value
for money: the Clenshaw-Curtis integration, or perhaps the FFT.  You
would have to recast your problem by doing some sort of substitution.

Perhaps post some latex code to show the exact type of integral you
are wanting to calculate.

Regards,

On May 5, 6:32 am, Achilleas Achilleos ma...@bristol.ac.uk wrote:
 Hi,

 I am wondering whether there exist any function in R (any package) that
 calculates Fourier Integrals.

 Particularly, I am interested for estimation of a Cosine Fourier integral...

 I would be much obliged if you could help me on this..

 Thanks.

 Andreas

 --

 __
 r-h...@r-project.org mailing listhttps://stat.ethz.ch/mailman/listinfo/r-help
 PLEASE do read the posting guidehttp://www.R-project.org/posting-guide.html
 and provide commented, minimal, self-contained, reproducible code.

__
R-help@r-project.org mailing list
https://stat.ethz.ch/mailman/listinfo/r-help
PLEASE do read the posting guide http://www.R-project.org/posting-guide.html
and provide commented, minimal, self-contained, reproducible code.


[R] Way to handle variable length and numbers of columns using read.table(...)

2009-05-04 Thread Jason Rupert

I've got read.table to successfully read in my table of three columns.  Most of 
the time I will have a set number of rows, but sometime that will be variable 
and sometimes there will be only be two variables in one row, e.g. 

Time Loc1 Loc2
1 22.33 44.55
2 66.77 88.99
3 222.33344.55
4 66.77 88.99

Is there any way to have read.table handle (1) a variable number of rows, and 
(2) sometime there are only two variables as shown in Time = 3 above? 

Just curious about how to handle this, and if read.table is the right way to go 
about or if I should read in all the data and then try to parse it out best I 
can.  

Thanks again.

 R.version
   _   
platform   i386-apple-darwin8.11.1 
arch   i386
os darwin8.11.1
system i386, darwin8.11.1  
status 
major  2   
minor  8.0 
year   2008
month  10  
day20  
svn rev46754   
language   R   
version.string R version 2.8.0 (2008-10-20)

__
R-help@r-project.org mailing list
https://stat.ethz.ch/mailman/listinfo/r-help
PLEASE do read the posting guide http://www.R-project.org/posting-guide.html
and provide commented, minimal, self-contained, reproducible code.


Re: [R] Way to handle variable length and numbers of columns using read.table(...)

2009-05-04 Thread jim holtman
Well if you read in your data, you get:

 x - read.table('clipboard', header=TRUE, fill=TRUE)
Warning message:
In read.table(clipboard, header = TRUE, fill = TRUE) :
  incomplete final line found by readTableHeader on 'clipboard'
 x
  Time Loc1  Loc2
1122.33 44.55
2266.77 88.99
33 222.33344.55NA
4466.77 88.99
 str(x)
'data.frame':   4 obs. of  3 variables:
 $ Time: int  1 2 3 4
 $ Loc1: Factor w/ 3 levels 22.33,222.33344.55,..: 1 3 2 3
 $ Loc2: num  44.5 89 NA 89

As you can see the variable that has two decimal points is read in as a
character and cause the whole column to be converted to a factor.  It
appears that you have some fixed length fields that are overflowing.  Now
you could read in the data and use regualr expressions and parse the data;
you just have to match on the first part have two decimal place and then
extract the rest.  THe question is, is this the only problems you have in
the data?  If so, parsing it is not hard.

On Mon, May 4, 2009 at 10:20 PM, Jason Rupert jasonkrup...@yahoo.comwrote:


 I've got read.table to successfully read in my table of three columns.
  Most of the time I will have a set number of rows, but sometime that will
 be variable and sometimes there will be only be two variables in one row,
 e.g.

 Time Loc1 Loc2
 1 22.33 44.55
 2 66.77 88.99
 3 222.33344.55
 4 66.77 88.99

 Is there any way to have read.table handle (1) a variable number of rows,
 and (2) sometime there are only two variables as shown in Time = 3 above?

 Just curious about how to handle this, and if read.table is the right way
 to go about or if I should read in all the data and then try to parse it out
 best I can.

 Thanks again.

  R.version
   _
 platform   i386-apple-darwin8.11.1
 arch   i386
 os darwin8.11.1
 system i386, darwin8.11.1
 status
 major  2
 minor  8.0
 year   2008
 month  10
 day20
 svn rev46754
 language   R
 version.string R version 2.8.0 (2008-10-20)

 __
 R-help@r-project.org mailing list
 https://stat.ethz.ch/mailman/listinfo/r-help
 PLEASE do read the posting guide
 http://www.R-project.org/posting-guide.htmlhttp://www.r-project.org/posting-guide.html
 and provide commented, minimal, self-contained, reproducible code.




-- 
Jim Holtman
Cincinnati, OH
+1 513 646 9390

What is the problem that you are trying to solve?

[[alternative HTML version deleted]]

__
R-help@r-project.org mailing list
https://stat.ethz.ch/mailman/listinfo/r-help
PLEASE do read the posting guide http://www.R-project.org/posting-guide.html
and provide commented, minimal, self-contained, reproducible code.


Re: [R] Way to handle variable length and numbers of columns using read.table(...)

2009-05-04 Thread Gabor Grothendieck
Its not clear exactly what the rules are for this but if we assume
that numbers always end in a decimal plus two digits then
using stapply from the gsubfn package:

 Lines - Time Loc1 Loc2
+ 1 22.33 44.55
+ 2 66.77 88.99
+ 3 222.33344.55
+ 4 66.77 88.99

 library(gsubfn)
 L - readLines(textConnection(Lines))
 strapply(L[-1], [0-9]*[.][0-9][0-9], as.numeric, simplify = rbind)
   [,1]   [,2]
[1,]  22.33  44.55
[2,]  66.77  88.99
[3,] 222.33 344.55
[4,]  66.77  88.99

See http://gsubfn.googlecode.com and for regular expressions see ?regex

On Mon, May 4, 2009 at 10:20 PM, Jason Rupert jasonkrup...@yahoo.com wrote:

 I've got read.table to successfully read in my table of three columns.  Most 
 of the time I will have a set number of rows, but sometime that will be 
 variable and sometimes there will be only be two variables in one row, e.g.

 Time Loc1 Loc2
 1 22.33 44.55
 2 66.77 88.99
 3 222.33344.55
 4 66.77 88.99

 Is there any way to have read.table handle (1) a variable number of rows, and 
 (2) sometime there are only two variables as shown in Time = 3 above?

 Just curious about how to handle this, and if read.table is the right way to 
 go about or if I should read in all the data and then try to parse it out 
 best I can.

 Thanks again.

 R.version
               _
 platform       i386-apple-darwin8.11.1
 arch           i386
 os             darwin8.11.1
 system         i386, darwin8.11.1
 status
 major          2
 minor          8.0
 year           2008
 month          10
 day            20
 svn rev        46754
 language       R
 version.string R version 2.8.0 (2008-10-20)

 __
 R-help@r-project.org mailing list
 https://stat.ethz.ch/mailman/listinfo/r-help
 PLEASE do read the posting guide http://www.R-project.org/posting-guide.html
 and provide commented, minimal, self-contained, reproducible code.


__
R-help@r-project.org mailing list
https://stat.ethz.ch/mailman/listinfo/r-help
PLEASE do read the posting guide http://www.R-project.org/posting-guide.html
and provide commented, minimal, self-contained, reproducible code.


Re: [R] Way to handle variable length and numbers of columns using read.table(...)

2009-05-04 Thread Jason Rupert

Jim, 

You guessed it.  There are other problems with the data.  Here is a closer 
representation of the data:
Total time and location 
are listed below.

Time Loc1 Loc2
---
1 22.33 44.55
2 66.77 88.99
3 222.33344.55
4 66.77 88.99

Avg. Loc1 = 77.88
Avg. Loc2 = 55.66
Final Time = 4

Right now I am using nrows in order to only read Time 1-4  skip to skip 
over the unusable header info, e.g.

read.table(read.table('clipboard', header=FALSE, fill=TRUE, skip=5, nrows=4)

Unfortunately, sometimes the number of Time rows varies, so I need to also 
account for that.  

Maybe I need to look into what Gabor suggested as well, i.e. library(gsubfn)

Thanks again for any feedback and advice on this one, as the data I receive is 
out of my control, but I am working with the go get them to fix it as well. 



--- On Mon, 5/4/09, jim holtman jholt...@gmail.com wrote:

 From: jim holtman jholt...@gmail.com
 Subject: Re: [R] Way to handle variable length and numbers of columns using  
 read.table(...)
 To: jasonkrup...@yahoo.com
 Cc: R-help@r-project.org
 Date: Monday, May 4, 2009, 9:47 PM
 Well if you read in your data, you get:
 
  x - read.table('clipboard', header=TRUE,
 fill=TRUE)
 Warning message:
 In read.table(clipboard, header = TRUE, fill =
 TRUE) :
   incomplete final line found by readTableHeader on
 'clipboard'
  x
   Time Loc1  Loc2
 1122.33 44.55
 2266.77 88.99
 33 222.33344.55NA
 4466.77 88.99
  str(x)
 'data.frame':   4 obs. of  3 variables:
  $ Time: int  1 2 3 4
  $ Loc1: Factor w/ 3 levels
 22.33,222.33344.55,..: 1 3 2 3
  $ Loc2: num  44.5 89 NA 89
 

As you can see the variable that has two decimal points is read in as a 
character and cause the whole column to be converted to a factor.  It appears 
that you have some fixed length fields that are overflowing.  Now you could 
read in the data and use regular expressions and parse the data; you just have 
to match on the first part have two decimal place and then extract the rest.  
THe question is, is this the only problems you have in the data?  If so, 
parsing it is not hard.
 
 On Mon, May 4, 2009 at 10:20 PM, Jason Rupert
 jasonkrup...@yahoo.comwrote:
 
 
  I've got read.table to successfully read in my
 table of three columns.
   Most of the time I will have a set number of rows,
 but sometime that will
  be variable and sometimes there will be only be two
 variables in one row,
  e.g.
 
  Time Loc1 Loc2
  1 22.33 44.55
  2 66.77 88.99
  3 222.33344.55
  4 66.77 88.99
 
  Is there any way to have read.table handle (1) a
 variable number of rows,
  and (2) sometime there are only two variables as shown
 in Time = 3 above?
 
  Just curious about how to handle this, and if
 read.table is the right way
  to go about or if I should read in all the data and
 then try to parse it out
  best I can.
 
  Thanks again.
 
   R.version
_
  platform   i386-apple-darwin8.11.1
  arch   i386
  os darwin8.11.1
  system i386, darwin8.11.1
  status
  major  2
  minor  8.0
  year   2008
  month  10
  day20
  svn rev46754
  language   R
  version.string R version 2.8.0 (2008-10-20)
 
  __
  R-help@r-project.org mailing list
  https://stat.ethz.ch/mailman/listinfo/r-help
  PLEASE do read the posting guide
 
 http://www.R-project.org/posting-guide.htmlhttp://www.r-project.org/posting-guide.html
  and provide commented, minimal, self-contained,
 reproducible code.
 
 
 
 
 -- 
 Jim Holtman
 Cincinnati, OH
 +1 513 646 9390
 
 What is the problem that you are trying to solve?

__
R-help@r-project.org mailing list
https://stat.ethz.ch/mailman/listinfo/r-help
PLEASE do read the posting guide http://www.R-project.org/posting-guide.html
and provide commented, minimal, self-contained, reproducible code.


Re: [R] Way to handle variable length and numbers of columns using read.table(...)

2009-05-04 Thread Gabor Grothendieck
The last line should be as follows (as the previous post missed the
time column).
The regular expression says either start from beginning (^) and look
for a string of digits, [0-9]+, or look for digits [0-9]*, a dot [.] and two
more digits [0-9][0-9].  Each time strapply finds such a match
as.numeric is applied to it.  Thus line of input results in a numeric
vector and then we simplify those vectors by rbind'ing them together.

 strapply(L[-1], ^[0-9]+|[0-9]*[.][0-9][0-9], as.numeric, simplify = rbind)
 [,1]   [,2]   [,3]
[1,]1  22.33  44.55
[2,]2  66.77  88.99
[3,]3 222.33 344.55
[4,]4  66.77  88.99


On Mon, May 4, 2009 at 11:04 PM, Gabor Grothendieck
ggrothendi...@gmail.com wrote:
 Its not clear exactly what the rules are for this but if we assume
 that numbers always end in a decimal plus two digits then
 using stapply from the gsubfn package:

 Lines - Time Loc1 Loc2
 + 1 22.33 44.55
 + 2 66.77 88.99
 + 3 222.33344.55
 + 4 66.77 88.99

 library(gsubfn)
 L - readLines(textConnection(Lines))
 strapply(L[-1], [0-9]*[.][0-9][0-9], as.numeric, simplify = rbind)
       [,1]   [,2]
 [1,]  22.33  44.55
 [2,]  66.77  88.99
 [3,] 222.33 344.55
 [4,]  66.77  88.99

 See http://gsubfn.googlecode.com and for regular expressions see ?regex

 On Mon, May 4, 2009 at 10:20 PM, Jason Rupert jasonkrup...@yahoo.com wrote:

 I've got read.table to successfully read in my table of three columns.  Most 
 of the time I will have a set number of rows, but sometime that will be 
 variable and sometimes there will be only be two variables in one row, e.g.

 Time Loc1 Loc2
 1 22.33 44.55
 2 66.77 88.99
 3 222.33344.55
 4 66.77 88.99

 Is there any way to have read.table handle (1) a variable number of rows, 
 and (2) sometime there are only two variables as shown in Time = 3 above?

 Just curious about how to handle this, and if read.table is the right way to 
 go about or if I should read in all the data and then try to parse it out 
 best I can.

 Thanks again.

 R.version
               _
 platform       i386-apple-darwin8.11.1
 arch           i386
 os             darwin8.11.1
 system         i386, darwin8.11.1
 status
 major          2
 minor          8.0
 year           2008
 month          10
 day            20
 svn rev        46754
 language       R
 version.string R version 2.8.0 (2008-10-20)

 __
 R-help@r-project.org mailing list
 https://stat.ethz.ch/mailman/listinfo/r-help
 PLEASE do read the posting guide http://www.R-project.org/posting-guide.html
 and provide commented, minimal, self-contained, reproducible code.



__
R-help@r-project.org mailing list
https://stat.ethz.ch/mailman/listinfo/r-help
PLEASE do read the posting guide http://www.R-project.org/posting-guide.html
and provide commented, minimal, self-contained, reproducible code.


Re: [R] Calculating relative ratios in a data frame..

2009-05-04 Thread Zeljko Vrba
On Mon, May 04, 2009 at 04:44:37PM -0400, jim holtman wrote:

 Here is a way -- you are dividing by zero in the scaling:
 
Thanks to both of you who answered.  I know about division by zero, but I
basically ignore rows having n==1 when plotting.

__
R-help@r-project.org mailing list
https://stat.ethz.ch/mailman/listinfo/r-help
PLEASE do read the posting guide http://www.R-project.org/posting-guide.html
and provide commented, minimal, self-contained, reproducible code.